Calcolo combinatorio

annuncio pubblicitario
Liceo Statale
Ettore Majorana
Roma
MAIORA QUAM LIBRI
Calcolo combinatorio
Dispensa per classi quarte del Liceo Scientifico
a cura del prof. Alexander Saltuari
Roma, 2016
1
• Sommario
 PREMESSA
1
 SEZIONE 1 – RAPPRESENTARE I CASI POSSIBILI
3
 LA RAPPRESENTAZIONE AD ALBERO
3
 CALCOLO DEI CASI POSSIBILI
4
 ALTRE CONSIDERAZIONI SULLA RAPPRESENTAZIONE AD ALBERO
6
 SOLUZIONI DEI PROBLEMI DELLA SEZIONE 1
9
 SEZIONE 2 – I RAGGRUPPAMENTI CLASSICI
11
 LE PERMUTAZIONI SEMPLICI
11
 LE PERMUTAZIONI CON RIPETIZIONE
12
 LE DISPOSIZIONI SEMPLICI
15
 LE DISPOSIZIONI GENERICHE SENZA RIPETIZIONE
16
 LE DISPOSIZIONI SEMPLICI CON RIPETIZIONE
17
 LE COMBINAZIONI SEMPLICI
19
 LE COMBINAZIONI CON RIPETIZIONE
24
 SOLUZIONI DEI PROBLEMI DELLA SEZIONE 2
31
 SEZIONE 3 – RISOLVERE I PROBLEMI
35
 PROBLEMI DI RIEPILOGO
39
 PROBLEMI DIFFICILISSIMI
43
 TEST DI AUTOVALUTAZIONE (SOLUZIONI A PAGINA 58)
45
 SOLUZIONI DEI PROBLEMI DELLA SEZIONE 3
48
 SOLUZIONI DEL TEST DI AUTOVALUTAZIONE
58
 SEZIONE 4 - INTRODUZIONE AL CALCOLO DELLE PROBABILITÀ
61
 GLI EVENTI E LA PROBABILITÀ
61
 METODI DI CALCOLO
64
 LA DISTRIBUZIONE BINOMIALE
68
 IL VALORE ATTESO
70
 SOLUZIONI DEI PROBLEMI DELLA SEZIONE 4
75
2
• Premessa
• Le dispense di Maiora Quam Libri
La presente dispensa si inserisce nella cornice dell’iniziativa scolastica Maiora Quam Libri
,
promossa dal Liceo Statale Majorana di Roma e mirata alla condivisione gratuita del materiale
didattico prodotto dagli insegnanti. I diritti sulla presente opera appartengono all’autore, in
questo caso Alexander Saltuari, e sono disciplinati dalla licenza di tipo CC BY-NC: è proibito
fare un uso commerciale del presente materiale, è invece permesso a chiunque scaricarlo,
copiarlo e modificarlo a patto di citarne la fonte, riconoscerne in modo chiaro la paternità e
fornire un link alla licenza CC BY-NC.
Il testo integrale della licenza è reperibile al sito https://creativecommons.org/licenses/by-ncsa/3.0/it/legalcode .
• Prerequisiti
I prerequisiti previsti per questa unità didattica sono poco più che elementari: il calcolo
numerico in ℤ e ℚ e, relativamente alla sezione 4 (Introduzione al calcolo delle probabilità),
gli insiemi. Il percorso proposto, ideato per una classe quarta del Liceo Scientifico, può quindi
essere affrontato senza modifiche in una classe prima.
• Come leggere il testo
La presente dispensa è frutto di una raccolta e di una risistemazione di materiale scolastico
prodotto nell’arco di 10 anni e quindi caratterizzato da stili tipografici e accenti matematici
differenti, adatti di volta in volta a classi diverse.
In questo testo si trovano spesso dei riquadri colorati. Essi hanno il seguente significato:
Nei riquadri blu stanno gli Esempi, comprensivi di soluzione. Si consiglia di leggere quest’ultima
soltanto dopo aver tentato di risolvere il problema autonomamente.
Nei riquadri gialli stanno i Problemi. Per capire i concetti e familiarizzare con le tecniche del
Calcolo Combinatorio è assolutamente necessario cercare di risolvere gli esercizi proposti. È del
tutto naturale fallire ai primi tentativi, per cui è importante non scoraggiarsi e perseverare. Le
soluzioni dei problemi sono poste in appendice a ciascuna sezione.
1
I riquadri verdi sono dedicati agli enunciati come Definizioni, Proposizioni, Teoremi.
I riquadri grigi contengono risultati parziali che devono ancora essere generalizzati.
Gli esercizi di riepilogo di fine unità, staccati dal resto del testo, non sono stati inseriti in alcun
riquadro colorato per non appesantire l’estetica complessiva.
2
• Sezione 1 – Rappresentare i casi possibili
Il calcolo combinatorio è quella parte della Matematica che si occupa dei modi per
raggruppare ed ordinare gli oggetti e del numero complessivo di tali raggruppamenti.
Supponiamo ad esempio di dover stabilire in quanti modi diversi si possa anagrammare la
parola “ASG”. Esempi possibili sono AGS, oppure GAS, oppure SAG, ecc. Nel contare le
combinazioni dobbiamo evitare di commettere i due errori più comuni, quali non considerare
tutte le possibili combinazioni e contare la stessa parola due o più volte. La prima cosa che
bisogna imparare per risolvere problemi di calcolo combinatorio è proprio la capacità di elencare
tutte le possibilità in modo ordinato. Una strategia molto efficace è la cosiddetta
rappresentazione ad albero.
• La rappresentazione ad albero
Cerchiamo di risolvere il problema esposto sopra e concentriamoci preliminarmente sulla prima
lettera della tripletta: evidentemente esistono tre possibilità (A, G e S) che si escludono a
vicenda.
A
G
S
Possiamo rappresentare questa situazione con lo schema
ad albero mostrato a fianco ( ← ) .
Ogni linea di questo schema è detta “ramo”, i punti neri
sono detti nodi e il nodo iniziale (a sinistra) è detto “radice”.
Quando la costruzione dell’albero è conclusa, i nodi finali (a
destra) vengono anche detti “foglie”.
Per iniziare consideriamo il ramo in alto, cioè supponiamo
di aver fissato come prima lettera la “A”. A questo punto
per la scelta della seconda lettera restano la “G” e la “S”.
Possiamo quindi “far crescere” il nostro albero come
illustrato a destra ( → ) .
A
A
G
S
G
S
G
S
G
A
S
S
Seguendo lo stesso ragionamento anche per i rami iniziali
“G” ed “S” si ottiene l’albero disegnato a fianco ( ← ) .
A
G
3
Continuiamo a far crescere l’albero e iniziamo dal
ramo superiore. Avendo già utilizzato due lettere,
la terza non può che essere la S. Notiamo che il
percorso che parte dalla radice e arriva fino al
ramo ora disegnato rappresenta la parola “finita”
AGS ( → ) .
Seguendo lo stesso procedimento disegniamo
l’intero albero fino ad ottenere il seguente schema
( → ) . Leggendo in ordine (dalla radice alla foglia)
i nomi dei nodi di ciascun percorso, si ottengono
le parole cercate.
A
G
S
S
G
A
S
S
A
G
A
G
S
G
S
S
G
A
S
S
A
A
G
G
A
In questo esempio, elencando dall’alto in basso, abbiamo trovato AGS, ASG, GAS, GSA, SAG,
SGA. Il numero di anagrammi della parola GAS è quindi 6 (che corrisponde al numero di foglie
dell’albero).
Questo modo di procedere può essere troppo dispendioso per gruppi di oggetti molto ampi. Il
vantaggio della rappresentazione ad albero risiede nella certezza che lo schema disegnato
contenga effettivamente tutte le possibilità e senza ripetizioni, ma, come vedremo fra poco,
l’albero delle possibilità si limita ad essere una schematizzazione ideale che spesso non deve
essere veramente costruita.
• Calcolo dei casi possibili
Per trovare una nuova strategia di calcolo adatta a problemi più complessi, cerchiamo di
analizzare il problema “GAS” in modo più astratto, senza elencare materialmente tutti i casi
possibili.
Per la scelta della prima lettera abbiamo tre possibilità. Qualunque lettera venga scelta essa non
potrà più essere usata. Questo vuol dire che per la seconda posizione restano aperte solo due
possibilità. Anche in questo caso, nel momento in cui la scelta è stata fatta e indipendentemente
dalla scelta stessa, resterà una sola lettera disponibile per occupare la terza posizione.
Riassumendo il discorso possiamo dire che
•
•
•
Per la prima posizione abbiamo tre possibili scelte
Qualsiasi sia stata la scelta precedente, per la seconda posizione esistono due possibili
scelte
Qualsiasi siano state le scelte precedenti, per la terza posizione (l’ultima ancora libera)
c’è ancora una sola possibilità
Per stabilire il numero di possibili “anagrammi” o modi di disporre le lettere, basta considerare
che dalla radice si diramano tre rami, che da ciascuno di essi partono altri due rami e che da
4
ciascuno di questi ultimi si dirama un ultimo ramo. Il numero di foglie (cioè di possibili percorsi) è
dato evidentemente dal prodotto 3 ⋅ 2 ⋅ 1 : il risultato è 6 , in accordo con quanto già trovato prima.
Notiamo che questo modo di procedere permette un calcolo veloce e non necessita di alcun
laborioso disegno. Come già accennato, si tratta di una schematizzazione ideale, nella quale
contiamo i rami senza disegnarli.
Se avessimo dovuto contare gli anagrammi della parola COSA, avremmo ragionato in maniera
identica, arrivando a 4 ⋅ 3 ⋅ 2 ⋅ 1 , gli anagrammi di ABITO ci avrebbero portato a 5 ⋅ 4 ⋅ 3 ⋅ 2 ⋅ 1 e così
via.
Applicando questo metodo, dovreste essere in grado di risolvere i seguenti problemi:
Esempio 1.1
- Gara dei 100m
In quanti modi diversi può finire una corsa con 8 concorrenti?
Soluzione
Naturalmente esistono 8 possibili vincitori. Fissato un vincitore esistono 7 possibili secondi classificati.
Fissati il primo e il secondo, esistono 6 possibili terzi e così via. Iterando il ragionamento otteniamo il
numero richiesto: 8 ⋅ 7 ⋅ 6 ⋅ 5 ⋅ 4 ⋅ 3 ⋅ 2 ⋅1 = 40.320 .
Da notare che se avessimo voluto disegnare anche solo le foglie dell’albero delle possibilità, avremmo
dovuto tratteggiare 40.320 nodi!
Esempio 1.2
- Assegnazione del podio
In quanti modi diversi possono essere assegnate le medaglie nella gara precedente?
Soluzione
Notiamo che in questo caso ci interessa solo sapere in quanti modi diversi possono essere occupate le
prime tre posizioni ed è irrilevante conoscere il quarto, il quinto o così via. Il modo di procedere è
praticamente identico a quello già visto:
Esistono 8 possibili vincitori. Fissato un vincitore esistono 7 possibili secondi classificati e fissati il primo
e il secondo esistono ancora 6 possibili terzi.
Il risultato è quindi 8 ⋅ 7 ⋅ 6 = 336 .
Esempio 1.3
- Rappresentanza sindacale
In un’azienda lavorano 24 operai, 8 impiegati e 3 addetti al trasporto. Bisogna costituire un comitato
formato da 1 rappresentante per categoria. In quanti modi è possibile formare il comitato?
Soluzione
Per gli operai esistono 24 possibili rappresentanti, per gli impiegati 8 e per i trasportatori 3. La soluzione è
quindi 24 ⋅ 8 ⋅ 3 = 576 .
5
Problema 1.1 - Assegnazione degli incarichi (soluzione a pagina 9)
In un’assemblea di 100 persone si devono scegliere un presidente e un segretario. Stabilire in quanti modi
è possibile effettuare la sceltaM
a) se gli incarichi sono fra loro incompatibili
b) se gli incarichi sono compatibili
Problema 1.2
- Il problema delle sedie (soluzione a pagina 9)
Ci sono 4 sedie libere, una di fianco all’altra. In quanti modi diverse possono essere occupate se ci sono
a) una persona
b) due persone
c) tre persone
d) quattro persone
e) cinque persone (una resta in piedi)
f)
sei persone (due restano in piedi)
Problema 1.3
- Estrazioni ordinate (soluzione a pagina 9)
In un’urna ci sono quattro palline contrassegnate dai numeri 1,2,3,4. Se si effettuano 3 estrazioni quanti
sono gli esiti possibili? (naturalmente va tenuto conto dell’ordine d’estrazione, per cui per esempio le
estrazioni 1,2,3 e 2,1,3 sono diverse fra loro)
Problema 1.4
- Il gruppo multiculturale (soluzione a pagina 9)
Un gruppo multiculturale è composto da 4 australiani, 4 brasiliani e 4 croati. All’interno del gruppo di 12
persone vanno designati un coordinatore, un supervisore e relatore, tenendo presente che le tre cariche
devono essere assunte da persone di nazionalità diversa (se ad esempio il supervisore fosse australiano
e il relatore croato, il coordinatore dovrebbe essere brasiliano). In quanti modi può essere effettuata la
scelta?
Problema 1.5
- X e Y (soluzione a pagina 10)
Si vuole formare una “parola” da 5 lettere utilizzando esclusivamente i simboli “X” e “Y” (per cui le parole
saranno del tipo YXXYX, YYYYY e così via). Quante parole del genere esistono?
• Altre considerazioni sulla rappresentazione ad albero
Analizziamo brevemente il passaggio logico che ci ha permesso, in molti degli esempi
precedenti, di ricondurre il conteggio di tutti i casi possibili al prodotto del numero dei rami di
ciascun livello. Come vedremo nei prossimi tre esempi, questo è consentito soltanto se l’albero
delle combinazioni ha una struttura regolare.
6
Esempio 1.4
- Gonne e magliette
Una signora ha nell’armadio quattro gonne e tre magliette.
Quanti sono gli abbinamenti possibili?
Soluzione
Possiamo rappresentare gli abbinamenti con una struttura
ad albero (vedi a fianco ( → ) ), immaginando di effettuare
prima la scelta relativa alle gonne e poi quella delle
magliette. Si tratta naturalmente di una schematizzazione
del tutto arbitraria, ciò che conta è che essa permette di
contare tutti gli abbinamenti possibili senza rischio di
contarne uno due volte.
Nell’illustrazione a fianco sono indicate con A,B,C,D le
quattro gonne e con 1,2,3 le tre magliette. La soluzione del
quesito è evidentemente 4 × 3 = 12 , perché a ciascuno dei
4 rami iniziali sono collegati gli stessi identici rami (cioè
1,2,3). Ciò suggerisce che, da un punto di vista
combinatorio, le scelte di gonna e di maglietta sono
indipendenti l’una dall’altra.
Il prossimo esempio è leggermente più complesso ma può essere risolto in maniera simile.
Esempio 1.5
- Capo e vice
Un gruppo di 4 ragazzi deve scegliere due rappresentanti, il
capo e il suo vice. In quanti modi è possibile effettuare la
scelta?
Soluzione
Ragionando come prima immaginiamo che la scelta venga
effettuata in due tempi: prima viene nominato il capo e poi il
vice (la scelta opposta sarebbe stata altrettanto corretta).
Stavolta la prima nomina influenza la seconda, come si vede
dall’albero a fianco: i tre rami superiori contengono le lettere
B,C,D mentre i tre rami sottostanti le lettere A,C,D. Si tratta
quindi di un problema più complesso del precedente (simile
al problema “GAS”). Ciononostante, il numero di rami di
“secondo livello” resta indipendente dal percorso (cioè dalla
scelta del capo), per cui, di nuovo, si può ridurre il calcolo al
prodotto 4 × 3 = 12 .
Ciò che conta è quindi l’indipendenza del numero delle
scelte, non delle scelte in quanto tali.
Per concludere questo tema vediamo un terzo esempio e due rappresentazioni ad esso
abbinate.
7
Esempio 1.6
- Il ragazzo fortunato
Un ragazzo trova per terra 3 biglietti validi per il cinema, relativi
a film diversi che verranno proiettati in giorni diversi. Il ragazzo
può quindi scegliere di vedere tutti e tre i film, di non vederne
nessuno o soltanto qualcuno. Quante scelte ha? Nota bene:
andare a vedere un film rispetto ad un altro è ovviamente una
scelta diversa.
Soluzione 1
Anche
stavolta
affrontiamo
il
problema
con
una
rappresentazione ad albero: essa ci permetterà di contare tutte
le opzioni una volta soltanto e quindi di arrivare alla soluzione.
Suddividiamo la scelta in due decisioni distinte:
1) Quanti film andare a vedere
2) Quali film andare a vedere
L’albero a fianco mostra che le opzioni sono complessivamente
8. A differenza dei due esempi precedenti, il numero di rami di
“secondo livello” è variabile, dipende cioè da quale primo ramo
si è percorso: in questo caso non è possibile ridurre il conteggio
a una semplice moltiplicazione del numero di diramazioni.
È importante sottolineare che la rappresentazione ad albero ha
comunque
permesso
un’utile
schematizzazione
e
si
è
dimostrata efficace .
Soluzione 2
Uno stesso problema può essere schematizzato in modi
molto diversi, come mostra la rappresentazione a fianco
( ← ) . Essa descrive lo stesso problema di prima (il
ragazzo fortunato), immaginando che la scelta avvenga
in tre fasi: per prima cosa il ragazzo decide se andare a
vedere il film A, successivamente se assistere alla
proiezione di B e soltanto alla fine se andare a vedere C.
Anche questo modo di vedere il problema assicura il
conteggio completo di tutte le possibili scelte. L’albero
che si crea è semplicissimo ( ← ) , e prevede ad ogni
diramazione lo stesso numero di opzioni. Il conteggio
può nuovamente essere ricondotto ad un prodotto, per la
precisione 2 × 2 × 2 = 8 , in accordo con quanto ottenuto
in precedenza.
Le strategie presentate fino qui non si applicano direttamente a tutti i tipi di problemi di calcolo
combinatorio che affronteremo. Ciononostante è bene avere una buona dimestichezza con tali
8
strumenti e tenere sempre a mente che il modo migliore per risolvere i problemi combinatori è
quello di trovare uno schema efficace per elencare tutti i casi possibili una volta soltanto.
• Soluzioni dei problemi della Sezione 1
Soluzione 1.1 - Assegnazione degli incarichi
a) Supponiamo che gli incarichi siano fra loro incompatibili. Ci sono 100 modi per scegliere il
presidente. Una volta eletto ci sono ancora 99 persone che possono ricoprire il ruolo di
segretario. La risposta è quindi 100 ⋅ 99 = 9900 .
b) Se gli incarichi sono compatibili (cioè il presidente può anche essere il segretario) per entrambe
le scelte ci sono 100 alternative. La risposta è quindi 100 = 10.000 .
2
Soluzione 1.2 - Il problema delle sedie
Chiamiamo le persone con le lettere dell’alfabeto: A, B, C ,...
a)
b)
A ha ovviamente 4 alternative.
A può scegliere fra 4 sedie. Una volta effettuata la scelta a B restano 3 alternative. La
soluzione è quindi 4 ⋅ 3 = 12 .
c) Ragionando come sopra A ha quattro possibilità, B tre e C due. Complessivamente ci sono
quindi 4 ⋅ 3 ⋅ 2 = 24 possibilità.
d) Questo problema è in verità identico a quello precedente (infatti una volta che A, B, C si sono
seduti D non ha che una scelta) e quindi la soluzione è 24.
e) Supponendo che si stia svolgendo una gara il cui primo premio è la prima sedia, il secondo
premio è la seconda e così via, il problema si riduce a calcolare il numero di possibili vincitori
moltiplicato per il numero di possibili secondi arrivati (una volta fissato il primo), e così via. In
sintesi la risposta è 5 ⋅ 4 ⋅ 3 ⋅ 2 = 120 .
f)
Possiamo ragionare come sopra è dire che le alternative sono 6 ⋅ 5 ⋅ 4 ⋅ 3 = 360 .
Osserviamo che in questo problema abbiamo implicitamente ammesso che le sedie e le persone fossero
fra loro distinguibili e che quindi per esempio le soluzioni A, B, C , D e D, C , A, B fossero considerate
distinte.
Soluzione 1.3 - Estrazioni ordinate
Ragionando come al solito possiamo dire che la prima estrazione può concludersi in 4 modi diversi, la
seconda in 3 (perché ora l’urna contiene solo 3 palline) e la terza 2. La soluzione è quindi 4 ⋅ 3 ⋅ 2 = 24 .
Soluzione 1.4 - Il gruppo multiculturale
Supponiamo di designare nell’ordine, il coordinatore, supervisore e il relatore.
Chiunque può essere eletto coordinatore, per cui vi sono 12 possibilità per la prima scelta.
Una volta stabilito il coordinatore, restano “in gioco” per la carica di supervisore, solo 8 persone (quelli
delle altre nazionalità).
Il titolo di relatore è appannaggio del gruppo nazionale non ancora rappresentato e composto da 4
persone. Riassumendo, la soluzione è 12 ⋅ 8 ⋅ 4 = 384 .
9
Soluzione 1.5 - X e Y
Scegliamo una lettera per volta. La prima lettera può essere una X o una Y (2 possibilità), la seconda
lettera, nuovamente, può essere una X o una Y (2 possibilità) e così via fino alla quinta lettera. La
soluzione è quindi
2 ⋅ 2 ⋅ 2 ⋅ 2 ⋅ 2 = 25 = 32 .
10
• Sezione 2 – I raggruppamenti classici
In questo capitolo tratteremo i sei raggruppamenti classici (classificati in permutazioni,
disposizioni e combinazioni). Essi rappresentano le strutture fondamentali del Calcolo
Combinatorio e costituiscono il nucleo di questa unità didattica.
• Le permutazioni semplici
Definizione
Dati n oggetti distinti, si chiama permutazione ogni riordino degli n oggetti.
Abbiamo già visto esempi di permutazione quando ci siamo occupati di anagrammi. E’
importante sottolineare che nelle permutazioni l’ordine degli oggetti gioca un ruolo
fondamentale. Questo ci permette di ragionare come abbiamo già fatto in precedenza: dato un
insieme di n elementi distinti ci sono n modi per occupare la prima posizione della sequenza,
dopodiché restano n − 1 modi per occupare la seconda posizione e così via, fino all’ultima
posizione che deve essere necessariamente occupata dall’unico elemento rimasto. Questo ci
porta a concludere cheM
Il numero di permutazioni in un insieme di n oggetti distinti è uguale a n ⋅ (n − 1) ⋅ (n − 2) ⋅ .... ⋅ 2 ⋅1
È opportuno riscrivere il risultato appena trovato introducendo un nuovo, importantissimo
simbolo:
Definizione
Per ogni intero positivo n , si chiama “ n fattoriale” e si indica con n! il prodotto
n ⋅ (n − 1) ⋅ (n − 2) ⋅ .... ⋅ 2 ⋅1 . Per definizione si pone 0! = 1 .
Esempio 2.1 - Fattoriale
Calcola 1!, 2! , 5! e 6!
Soluzione
Per definizione si ha 1! = 1 , 2! = 2 ⋅ 1 = 2 e
5! = 5 ⋅ 4 ⋅ ... ⋅ 2 ⋅1 = 120 . Per calcolare 6! osserviamo che
6! = 6 ⋅ ( 5!) = 6 ⋅120 = 720 .
Numero di permutazioni
Il numero di permutazioni in un insieme di n oggetti distinti è uguale a n!
Esempio 2.2
- Anagrammi di Amore
Quanti anagrammi della parola “Amore” esistono?
11
Soluzione
Si tratta di contare il numero di permutazioni di una sequenza di 5 lettere distinte. La soluzione è 5! = 120
Esempio 2.3
- Mischiare 40 carte
In quanti modi diversi può essere mischiato un mazzo di carte da briscola (40 carte)?
Soluzione
Si tratta di contare le possibili permutazioni del mazzo di carte. La soluzione quindi non è altro che 40! .
Notiamo
che
tale
numero
è
enorme
e
1.000.000.000.000.000.000.000.000.000.000.000.000.000.000.000.000 .
E’
vale
quindi
suppergiù
abbastanza
improbabile che due persone giochino casualmente la stessa partita due volte.
Quest’ultimo esempio fornisce una nuova chiave interpretativa: il numero di permutazioni è
uguale al numero di “mischiate” di un certo insieme (sempre che esso abbia elementi tutti diversi
fra loro).
Problema 2.1 - Metal Detector (soluzione a pagina 31)
5 amici devono passare, una alla volta, attraverso un metal detector. In quanti modi possono mettersi in
fila?
Problema 2.2 - Una lettrice accanita (soluzione a pagina 31)
Per il suo compleanno Annamaria ha ricevuto ben 4 libri diversi e ha deciso di leggerne uno la settimana
per un mese. In quanti modi può scegliere l’ordine di lettura?
Problema 2.3 - Numeri e lettere (soluzione a pagina 31)
a) Quanti sono gli anagrammi della “parola” “ABCD1234”, tali che il primo simbolo sia una lettera, il
secondo un numero, il terzo una lettera e così via in senso alternato?
b) Quanti anagrammi della parola “ABCD1234” hanno lettere e numeri alternati? Nota bene: rispetto
al precedente manca la condizione che il primo simbolo sia una lettera.
Problema 2.4 - Anagramma con restrizione/1 (soluzione a pagina 31)
Quanti anagrammi della parola “MARCO” hanno le lettere R e C affiancate (come MARCO o CROMA)?
Problema 2.5 - Anagramma con restrizione/2 (soluzione a pagina 31)
In quanti anagrammi della parola “OSTIA” le due consonanti non sono affiancate (come per esempio
OASIT o TAIOS)?
• Le permutazioni con ripetizione
Affrontiamo ora un caso particolare: in quanti modi è possibile anagrammare la parola BAAA?
Applicando acriticamente la formula vista precedentemente, la risposta sembrerebbe essere 24
(cioè 4! ) , mentre è evidente che gli unici anagramma di BAAA sono BAAA stesso, ABAA,
12
AABA e AAAB. Questa apparente contraddizione deriva dal fatto che la collezione di lettere B,
A, A, A non è composta da oggetti distinti. Abbiamo così constatato che la ripetizione di un
elemento nella collezione di oggetti rende la formula delle permutazioni semplici inapplicabile.
Definizione
Data una collezione di n oggetti, alcuni dei quali possono essere ripetuti, si chiama
permutazione con ripetizione ogni riordino della collezione.
Esempi di collezioni “con ripetizione”: anagrammi di “MATEMATICA”, le carte da gioco di due
mazzi mischiati insieme, l’inventario di un negozio di abbigliamento e così via.
Per calcolare il numero di permutazioni di questo genere di collezioni, chiediamoci innanzitutto
qual è la causa dell’errore nell’applicare la formula per le permutazioni semplici. Riprendiamo
l’esempio della parola BAAA e aggiungiamo per comodità un pedice alle “A”, in modo da poterle
distinguere. Abbiamo quindi la parola BA1A2A3 (composta ora da oggetti quattro diversi). Grazie
a questa “differenziazione forzata”, sappiamo, tra le altre cose, che esistono 6 diversi
anagrammi che iniziamo con la “B”, per la precisione BA1A2A3, BA1A3A2, BA2A1A3, BA2A3A1,
BA3A2A1, BA3A1A2. (abbiamo contato il numero di “mischiate” delle diverse “A” tenendo fissa la
“B”). Analogamente esisteranno 6 diversi anagrammi con la “B” al secondo posto, 6 anagrammi
con la “B” al penultimo posto e altrettanti con la “B” alla fine.
È bene ricordare che i pedici introdotti sono posticci e che essi non esistono in originale: le “A”
sono fra loro a tutti gli effetti indistinguibili. Applicando la formula classica facciamo quindi
l’errore di contare per ben 6 volte BAAA e analogamente, di contare 6 volte ABAA, AABA e
AAAB. Visto che ogni anagramma viene contato esattamente 6 volte, la soluzione corretta si
ottiene dividendo il numero di anagrammi di BA1A2A3 per 6. Così si ha 24 / 6 = 4 , come era già
stato mostrato.
Riassumiamo il ragionamento: quando un elemento di una collezione si ripete, dobbiamo
dividere il numero di permutazioni semplici per il numero delle possibili “mischiate” degli
elementi ripetuti. Applicando quindi la formula già presentata possiamo concludere cheM
Il numero di permutazioni in una collezione di n oggetti, in cui vi è un (solo) oggetto che compare
k volte, è uguale a
n!
.
k!
Si tratta di un risultato soltanto parziale, perché consente di affrontare soltanto il caso in cui le
ripetizioni riguardano un solo elemento della collezione.
13
Esempio 2.4
- Anagrammi di Torta
Calcolare il numero di anagrammi possibili della parola TORTA.
Soluzione
Notiamo che la parola TORTA è composta da oggetti distinti, eccezion fatta per la T che compare 2 volte.
Possiamo quindi applicare la formula appena vista e concludere che la risposta è
Esempio 2.5
5! 120
=
= 60 .
2!
2
Anagrammi di Mamma
Calcolare il numero di anagrammi possibili della parola MAMMA.
Stavolta le ripetizioni riguardano non una, ma due lettere e la formula
n!
è inadeguata.
k!
Cerchiamo dunque di generalizzare il procedimento applicato prima e differenziamo le lettere
uguali con dei pedici. La parola (formata ora da lettere distinte) M1A1M2M3A2 ha 5! = 120
permutazioni. Mischiando fra loro le M1, M2 e M3 otteniamo parole che, senza pedici, sarebbero
indistinguibili. Stesso effetto scambiando fra loro A1 e A2. Una volta tolti i pedici abbiamo quindi
2!⋅ 3! versioni della stessa parola. La soluzione del problema è allora
5!
5 ⋅ 4 ⋅ 3 ⋅ 2 ⋅1
=
= 10 .
2!3! 1 ⋅ 2 ⋅1 ⋅ 2 ⋅ 3
La generalizzazione del procedimento visto è immediata e porta alla seguente formula, stavolta
definitiva:
Numero di permutazioni con ripetizione
Il numero di permutazioni in una collezione di n oggetti, in cui un certo oggetto compare k1
volte, un altro compare k2 volte e così via per r oggetti, è uguale a
Esempio 2.6
n!
.
k1 !⋅ k 2 !⋅ ... ⋅ kr !
- Anagrammi di Combinatorio
Calcolare il numero di anagrammi possibili della parola COMBINATORIO.
Soluzione
Nella parola di 12 lettere COMBINATORIO, la “O” compare 3 vote e la “I” 2. Il numero di anagrammi è
quindi
12!
= 39.916.800 .
3!⋅ 2!
Problema 2.6 - Anagrammi di Matematica (soluzione a pagina 31)
Calcola il numero di anagrammi della parola MATEMATICA.
Problema 2.7 - Calcoli su Calcolo (soluzione a pagina 31)
Quanti anagrammi della parola CALCOLO hanno le due “c” affiancate?
14
Problema 2.8 - Un piccolo robot (soluzione a pagina 32)
Un piccolo robot si muove percorrendo sempre e solo tratti rettilinei di 1 metro in avanti o indietro. Quanti
itinerari diversi composti da 8 tratti consecutivi riportano il robot al punto di partenza?
• Le disposizioni semplici
Definizione
Si chiama disposizione una sequenza ordinata di k oggetti distinti estratti da una collezione di n
oggetti distinti (per cui k ≤ n ). Se l’unica condizione posta è quella della non–ripetibilità degli
elementi all’interno della sequenza, la disposizione è detta semplice.
La differenza fra disposizioni semplici e permutazioni è che nelle disposizioni semplici non
vengono utilizzati tutti gli oggetti dell’insieme. L’esempio Assegnazione del podio visto in
precedenza è un caso tipico di disposizione semplice. Notiamo che anche nelle disposizioni
appare il concetto di ordine e quindi il modo di procedere è analogo a quello delle permutazioni.
Supponiamo di dover contare il numero di disposizioni semplici di k oggetti di un insieme di n
oggetti. Per occupare la prima posizione vi sono n possibilità, dopodiché abbiamo ancora n − 1
modi per occupare la seconda posizione (visto che l’unica condizione posta è che gli elementi
siano diversi fra loro), n − 2 per occupare la terza e così via per k volte. Questo ci porta a
concludere che
Il numero di disposizioni semplici di k oggetti in un insieme di n oggetti distinti è uguale a
n ⋅ (n −1) ⋅ (n − 2) ⋅ .... ⋅ (n − k + 1) .
Nel calcolo combinatorio è consuetudine utilizzare quando possibile il simbolo di fattoriale. Un
breve calcolo mostra la seguente identità:
n ⋅ (n − 1) ⋅ (n − 2) ⋅ .... ⋅ (n − k + 1) =
n ⋅ (n − 1) ⋅ (n − 2) ⋅ .... ⋅ (n − k + 1)(n − k ) ⋅ (n − k − 1) ⋅ ... ⋅ 2 ⋅1
n!
=
(n − k ) ⋅ (n − k − 1) ⋅ ... ⋅ 2 ⋅1
(n − k )!
.
Possiamo quindi sostituire la proposizione precedente con la seguente:
Proposizione
Il numero di disposizioni semplici di k oggetti in un insieme di n oggetti distinti è uguale a
n!
(n − k )!
Esempio 2.7
- Parole senza ripetizioni
Quante “parole” di 5 lettere fra loro distinte si possono formare con l’alfabeto italiano?
Soluzione
Abbiamo 21 modi (il numero di lettere dell’alfabeto italiano) per scegliere la lettera iniziale, 20 per
occupare la seconda posizione (perché la seconda lettera deve essere diversa dalla prima), 19 per la
15
terza (perché la terza deve essere diversa dalle due precedenti) e così via. Il risultato sarà quindi
21 ⋅ 20 ⋅19 ⋅18 ⋅17 = 2.441.880 . Volendo utilizzare la formula
21!
=
21!
(21 − 5)! 16!
=
51090942171709440000
20922789888000
n!
( n − k )!
otterremmo
di
nuovo
= 2.441.880 .
Come si vede nello svolgimento dell’ultimo esempio, la formula
n!
è elegante ma poco
(n − k )!
pratica, poiché coinvolge numeri enormi, destinati comunque ad essere riassorbiti da qualche
semplificazione. Il modo migliore per affrontare le disposizioni, semplici e non, è quello visto nel
capitolo introduttivo: il calcolo dei casi possibili.
Esempio 2.8
- Numeri senza ripetizioni
Quanti numeri naturali composti da 5 cifre diverse fra loro e diverse da 0 esistono?
Soluzione
Come al solito, avendo a che fare con una sequenza ordinata, conviene fissare un simbolo per volta,
partendo dalla cifra più a sinistra.
Per la prima cifra si hanno 9 possibili scelte: “1”,”2”,”3”,M,”9”. Qualsiasi sia la cifra prescelta, essa non
potrà più essere usata. Per la seconda cifra restano quindi 8 possibilità. Ragionando in questo modo,
arriviamo subito alla soluzione 9 ⋅ 8 ⋅ 7 ⋅ 6 ⋅ 5 = 15.120 .
• Le disposizioni generiche senza ripetizione
Ripercorriamo il modo di procedere nel calcolo delle disposizioni: per effettuare la prima scelta
(generalmente l’elemento in prima posizione) abbiamo un certo numero m1 di possibili opzioni,
per effettuare la seconda scelta possiamo abbiamo m2 alternative e così via per k volte. In
generale il numero totale di disposizioni sarà dato dal prodotto di questi numeri, cioè
D = m1 ⋅ m2 ⋅ ... ⋅ mk . Il calcolo nei vari mi modi non sempre è riconducibile a quello delle
disposizioni semplici, cioè non possiamo porre sempre m1 = n , m2 = n − 1, ecc. Vediamo a
questo proposito un esempio:
Esempio 2.9
- Il podio multinazionale
Tre corridori spagnoli ( S1 , S2 , S3 ), tre tedeschi ( T1 , T2 , T3 ) e tre francesi ( F1 , F2 , F3 ) partecipano a una
gara. Quanti possibili “podi” vedono tutte e tre le nazioni rappresentate?
16
Soluzione
Gli atleti in gara sono 9 e quindi esistono 9 modi per assegnare la medaglia d’oro. Fissato il vincitore
esistono 6 possibili secondi (perché dobbiamo escludere i connazionali del vincitore) e, seguendo lo
stesso procedimento, tre possibili terzi (uno qualunque della nazione non ancora premiata). Possiamo
quindi concludere che la risposta è 9 ⋅ 6 ⋅ 3 = 162 .
In generale, qualsiasi problema risolto con il metodo sopra descritto, cioè quello di considerare il
numero delle possibili scelte secondo un preciso ordine (normalmente è un ordine che il
problema stesso suggerisce) e di moltiplicare tali valori fra loro, è una disposizione.
• Le disposizioni semplici con ripetizione
In tutti i casi visti finora, se un elemento occupava una certa posizione non poteva occuparne
altre: in senso figurato, ogni oggetto poteva essere “usato” una sola volta. In molti problemi
questa esclusività non esiste, come si evince dal seguente esempio (molto simile ad un
problema visto in precedenza):
Esempio 2.10 - Numeri con ripetizioni
Quanti numeri naturali composti da 5 cifre non nulle esistono (come 16.378 o 22.525 ) ?
Soluzione
Abbiamo a che fare con una sequenza ordinata e questo ci permette di fissare una cifra alla volta. Per la
prima posizione si hanno 9 possibili scelte: “1”,”2”,”3”,M,”9”. Il problema non pone alcuna condizione circa
la ripetibilità delle cifre, per cui si hanno 9 possibilità anche per la seconda, la terza, la quarta e l’ultima
cifra. La soluzione è quindi 9 ⋅ 9 ⋅ 9 ⋅ 9 ⋅ 9 = 9 = 59.049 .
5
In generale, se ogni elemento può essere riusato, la soluzione sarà uguale al prodotto ripetuto di
una certa costante per se stessa (infatti ad ogni passo si ha lo stesso numero di scelte). In altre
parole, il numero di permutazioni con ripetizioni è una potenza.
Esempio 2.11 - Parole di tre lettere
Usando soltanto lettere dell’alfabeto italiano, quante parole (anche senza significato) di tre lettere possono
essere composte?
Soluzione
L’alfabeto italiano consta di 21 lettere. Ciascuna posizione può quindi essere occupata in 21 modi diversi
e la soluzione è 21⋅ 21⋅ 21 = 21 = 9261 .
3
La semplicità delle disposizioni con ripetizione è evidente. Presento quindi subito definizione e
metodo di calcolo:
17
Definizione
Si chiama disposizione con ripetizione una sequenza ordinata di k oggetti scelti da una
collezione di n oggetti distinti.
Numero di disposizioni con ripetizione.
n
Il numero di disposizioni con ripetizione di k oggetti scelti da una collezione di n oggetti è k .
Concludiamo la parte relativa alle disposizioni semplici con ripetizione con un esempio di
disposizione con ripetizione che però esula dalla definizione data sopra (e infatti, a rigore, non si
tratta di una disposizione “semplice”)
Esempio 2.12 – Un “TOT”
Usando l’alfabeto italiano, quante parole di tre lettere si possono formare, tali che la prima e l’ultima lettera
siano consonanti e la centrale sia una vocale (come “TOT” o “PER”)?
Soluzione
Il modo di ragionare è sempre quello delle disposizioni semplici con ripetizione, cambia soltanto la
collezione “di pesca”: per la prima lettera vi sono 16 possibilità (numero di consonanti dell’alfabeto
italiano), per la seconda posizione si hanno 5 scelte (vocali possibili) e per scegliere l’ultima consonante vi
sono nuovamente 16 opzioni. La soluzione è quindi 16 × 5 × 16 = 1280 .
Problema 2.9 - Lucchetto a combinazione (soluzione a pagina 32)
Un certo lucchetto a combinazione è composto da 4 rotelle, ciascuna delle quali può esser fatta scorrere a
indicare una delle 10 cifre 0,1,2,M9. Ovviamente il lucchetto si apre solo se tutte e quattro le rotelle sono
nella posizione corretta. Andando a caso, quale è il numero massimo di tentativi necessari per trovare la
combinazione corretta?
Problema 2.10 - Targhe italiane (soluzione a pagina 32)
a) Le targhe delle automobili in Italia hanno la forma L L N N N L L , dove le L sono le
lettere di un alfabeto anglo-italiano a 22 lettere e N
sono cifre del sistema decimale. Quante
possibili targhe possono essere create con questo sistema?
b) Per evitare fraintendimenti, nessuna targa italiana comincia con la coppie di lettere “EE” (in
pratica si è passati da ED999ZZ a EF000AA). Considerando questa limitazione, quante possibili
targhe automobilistiche italiane possono essere creare?
Problema 2.11 - Invito per 7 (soluzione a pagina 32)
7 persone, indicate con A, B, C, D, E, F, G, ricevono un invito a cena. Potrebbero andare tutti, potrebbe
non andare nessuno, potrebbero andare soltanto A e G o soltanto D, F e G e così via. Quante possibilità
ci sono?
18
• Le combinazioni semplici
Fino ad ora abbiamo visto soltanto casi in cui l’ordine degli elementi giocava un ruolo
determinante. Consideriamo adesso un problema di natura diversa:
Esempio 2.13 - Vendita di biciclette
Una persona possiede 5 biciclette diverse (indicate con A, B, C, D, E) e decide di venderne 3. In quanti
modi diversi può scegliere?
Notiamo che in questo caso l’ordine è irrilevante: se la persona vendesse A,C,D oppure C,A,D
farebbe la stessa identica scelta. Questo tipo di problema si riconduce a un problema relativo
alle cosiddette combinazioni.
Definizione
Si chiama combinazione (semplice) di k elementi di un insieme di n oggetti, un gruppo di k
elementi distinti estratto in un ordine qualsiasi dall’insieme. Il numero di combinazioni di k
elementi estratti da un insieme di n oggetti si indica spesso con C n ,k .
Esempio: nel SuperEnalotto© si parla opportunamente di “combinazione vincente” e non di
“disposizione vincente”. L’estrazione dei 6 numeri vincenti, poniamo 10, 20, 30, 40, 50, 60, può
infatti avvenire in un ordine qualsiasi.
Torniamo ora al problema delle biciclette e vediamo di risolvere la questione:
Soluzione del problema Vendita di biciclette
Sorprendentemente possiamo ricondurre la questione al conteggio degli anagrammi (di cui oramai
conosciamo tutti i segreti). Possiamo infatti descrivere ogni scelta “di vendita” in modo univoco utilizzando
una sequenza di 5 simboli. La sequenza
A,B,C, mentre
V
X
V
V
X
V
V
V
X
X
indicherebbe per esempio la vendita delle bici
la vendita di A,C,D.
Le possibili scelte di riducono quindi agli anagrammi della parola (con ripetizione) “VVVXX”. La soluzione
è
5!
3! × 2!
, perché le uniche due lettere
V
e
X
si ripetono una 3 e l’altra 2 volte.
A prima vista potrebbe non risultare chiaro se la strategia da noi adottata abbia validità generale
o meno. Cerchiamo quindi di risolvere un altro problema riguardante le combinazioni.
Esempio 2.14 - Delegazione di lavoratori
6 lavoratori di una azienda con 15 dipendenti devono comporre una piccola delegazione. Quante
delegazioni diverse possono essere create?
19
Soluzione
Disponendo i lavoratori in ordine alfabetico, potremmo descrivere ogni possibile delegazione barrando su
un
foglio
X
6
caselle
X X
su
X
15.
Così
ad
X X
esempio
la
sequenza
rappresenterebbe in modo univoco i nomi
dei 6 dipendenti prescelti. Considerando per semplicità delle O al posto delle caselle rimaste vuote, si
tratta di contare gli anagrammi della parola XXXXXXOOOOOOOOO . La soluzione è
6 volte
9 volte
15!
6! ⋅ 9!
= 5005 .
Evidentemente la strategia funziona. Possiamo facilmente generalizzare il risultato a una classe
composta da n studenti da cui “estrarre” una delegazione di k ragazzi (ovviamente deve
essere k ≤ n ). La parola da anagrammare avrà n lettere, k delle quali saranno delle
X
e
n − k delle O . Abbiamo così trovato il seguente risultato
Per le combinazioni semplici vale la formula Cn ,k =
n!
.
k ! ⋅ (n − k )!
Vista l’importanza di questo risultato, esiste in matematica un simbolo che rappresenta la
quantità
n!
, detta anche coefficiente binomiale:
k ! ⋅ (n − k )!
n
n!
e si legge “n su k”.
 =
 k  k ! ⋅ (n − k )!
Bisogna prestare attenzione al fatto che non c’è il simbolo di frazione (la lineetta) nel simbolo
n
  . Notiamo che, per definizione, 0! = 1 e quindi
k 
n
n
n!
n!
n!
n!
n!
= = 1 e anche   =
=
= = 1 . Ragionando sul senso
 =
 0  0! ⋅ (n − 0)! n !
 n  n ! ⋅ (n − n)! n ! ⋅ 0! n !
n
n
combinatorio di   possiamo senz’altro affermare che   è definito se 0 ≤ k ≤ n .
k 
k 
del
coefficiente
binomiale
RiassumiamoM
Definizione di Coefficiente Binomiale
n
k 
Dati due interi k, n con 0 ≤ k ≤ n , si definisce il simbolo   =
n!
.
k ! ⋅ (n − k )!
Enunciamo ora di nuovo la proposizione con la nuova simbologia:
20
Teorema sulle combinazioni
Il numero di combinazioni composte da k oggetti diversi di un insieme di n oggetti distinti è
n
k 
uguale a   .
I coefficienti binomiali hanno parecchie caratteristiche che qui non elencherò. L’unico “trucco”
che voglio citare facilita il calcolo del coefficiente stesso (questo “trucco” in verità può essere
 79 
 . Non
4
facilmente ricavato dai calcoli visti per le disposizioni): supponiamo di voler calcolare 
è pensabile utilizzare la definizione e calcolare
79!
perché i numeri in gioco sono troppo
4!⋅ 75!
grandi. Basta però sapere che il risultato è sempre dato da una frazione così composta:
•
•
nel numeratore compaiono k fattori, che partono da n e scendono di una unità
anche nel denominatore compaiono k fattori, che partono da 1 e crescono di una unità
 79  79 ⋅ 78 ⋅ 77 ⋅ 76
(ci sono 4 fattori sia nel
=
1⋅ 2 ⋅ 3 ⋅ 4
4
Seguendo questa definizione posso dire che 
numeratore che nel denominatore, in un caso decrescenti – partendo da 79 - e nell’altro
crescenti – partendo da 1) e quindi posso calcolare facilmente il numero
79 ⋅ 78 ⋅ 77 ⋅ 76
= 1.502.501 .
1⋅ 2 ⋅ 3 ⋅ 4
Problema 2.12 - Coefficienti binomiali (soluzione a pagina 32)
Calcolare con il metodo appena descritto e senza l’uso della calcolatrice i valori dei seguenti coefficienti
binomiali:
10  12 
 4 ,  1 ,
   
7
5
 
Esempio 2.15 - Due gruppi diversi
Una classe composta da 12 alunni deve dividersi in due gruppi rispettivamente da 5 e da 7 persone. In
quanti modi è possibile effettuare tale scelta?
Soluzione
Notiamo che ogni possibile scelta del primo gruppo (e anche del secondo) è per definizione una
combinazione di 5 elementi (nel secondo di 7) di un insieme composto da 12 elementi. E’ evidente che
scegliendo uno dei due gruppi automaticamente sarà determinato anche l’altro. Il risultato è quindi
 12  12 ⋅ 11 ⋅ 10 ⋅ 9 ⋅ 8
 5  = 1 ⋅ 2 ⋅ 3 ⋅ 4 ⋅ 5 = 792 . E’ facile verificare che anche partendo dalla scelta del secondo gruppo il
 
21
risultato resta immutato:
 12  12 ⋅ 11 ⋅ 10 ⋅ 9 ⋅ 8 ⋅ 7 ⋅ 6 12 ⋅ 11 ⋅ 10 ⋅ 9 ⋅ 8 ⋅ 7 ⋅ 6
 7  = 1 ⋅ 2 ⋅ 3 ⋅ 4 ⋅ 5 ⋅ 6 ⋅ 7 = 1 ⋅ 2 ⋅ 3 ⋅ 4 ⋅ 5 ⋅ 6 ⋅ 7 = 792 .
 
Se non ci fossimo accorti che la scelta del primo gruppo implicava la scelta dell’altro, avremmo
dovuto calcolare il numero di possibili gruppi da 7 persone che si potevano formare con gli
studenti rimasti liberi. Visto che gli studenti non ancora assegnati dopo la scelta del primo
7
gruppo erano 12 − 5 = 7 , tale quantità era   = 1 . Il numero totale di combinazioni sarebbe
7
 12   7 
 ⋅   = 792 .
 5  7
quindi rimasto 
Il metodo usato in quest’ultimo calcolo suggerisce che il problema dei Due gruppi diversi non è
nient’altro che una disposizione: abbiamo infatti calcolato in ordine le possibili combinazioni
prima di un gruppo e poi dell’altro moltiplicandole infine assieme. Per calcolare i singoli valori
abbiamo dovuto far uso delle combinazioni (cioè delle disposizione senz’ordine), il che fa di
questo problema il primo esempio “ibrido” visto finora: una disposizione di combinazioni.
Esempio 2.16 - Tre gruppi uguali ma non troppo
Una classe composta da 12 alunni deve dividersi in tre gruppi da 4 elementi e ad ogni gruppo viene
assegnato un lavoro diverso. In quanti modi è possibile effettuare tale scelta?
Soluzione
Notiamo che in questo caso, scelto un gruppo, non è più vero che gli altri gruppi sono automaticamente
definiti. Per prima cosa calcoliamo in quanti modi possibili posso scegliere il primo gruppo. Si tratta di una
 12  12 ⋅ 11 ⋅ 10 ⋅ 9
 4  = 1 ⋅ 2 ⋅ 3 ⋅ 4 = 495 . Restano
 
8 8⋅7⋅6⋅5
a disposizione soltanto 8 alunni e quindi, per scegliere il prossimo gruppo, abbiamo   =
= 70
 4  1⋅ 2 ⋅ 3 ⋅ 4
combinazione di 4 elementi in un insieme di 12 e quindi la risposta è
possibilità. Ormai anche il terzo gruppo è implicitamente definito (infatti volendo calcolare le scelte
possibili risulta che
 4  4 ⋅ 3 ⋅ 2 ⋅1
12   8   4 
 4  = 1 ⋅ 2 ⋅ 3 ⋅ 4 = 1 ) e quindi la soluzione è  4  ⋅  4  ⋅  4  = 495 ⋅ 70 ⋅ 1 = 34.650 .
 
     
Notiamo che nell’esempio appena visto ad ogni gruppo era assegnato un lavoro diverso. Se
invece i tre gruppi fossero stati fra loro indistinguibili (pensato al caso dello stesso lavoro di
gruppo) la soluzione trovata sarebbe stata sbagliata. Cerchiamo brevemente di spiegare il
perché e supponiamo che i tre gruppi si riuniscano in tre zone diverse dell’aula, a sinistra, in
centro e a destra. Chiamiamo A , B e C i tre diversi gruppi di lavoro. E’ evidente che una
diversa disposizione dei tre gruppi all’interno dell’aula non può essere considerata come una
suddivisione realmente diversa. In altre parole, calcolando la disposizione di combinazioni di
22
studenti, A, B, C e B, A, C vengono per esempio contati entrambi. E’ chiaro che ogni
disposizione di gruppi viene contata 6 volte (le possibili “mischiate” all’interno dell’aula) e che
12   8   4 
 ⋅ ⋅ 
4
4 4
34.650
quindi il risultato corretto in questo caso sarebbe       =
= 5775 .
3!
6
Vediamo un altro esempio che ci permetterà di vedere sia disposizioni che combinazioni:
Esempio 2.17 - 2 ragazze e 2 ragazzi
Un gruppo di amici è formato da due ragazze (Alice e Beatrice) e due ragazzi (Claudio e Davide).
a) Quante coppie maschio-femmina possono essere costituite?
b) Quante coppie generiche (cioè composte da due persone) possono essere formate?
c) In quanti modi il gruppo può dividersi in coppie maschio-femmina?
d) In quanti modi il gruppo può dividersi in coppie generiche?
Soluzione
a) Formiamo ogni coppia scegliendo prima la ragazza e poi il ragazzo. Per il primo caso si hanno 2
possibilità, così anche per il secondo. Riassumendo esistono 2 ⋅ 2 = 4 coppie femmina-maschio
(precisamente ( A, C ) , ( A, D) , ( B, C ) , ( B, D) ).
b) Si tratta di trovare il numeri di modi per scegliere due elementi in un insieme di 4. La risposta è
 4 4⋅3
 2  = 1 ⋅ 2 = 6 . Le possibili coppie sono ( A, B) , ( A, C ) , ( A, D) , ( B, C ) , (B, D) , (C, D) .
 
c) Rispetto al primo problema il numero sarà sicuramente minore, perché il gruppo non può
suddividersi in tutte le coppie possibili. Per esempio la suddivisione del gruppo in ( A, C ) e
( B, C )
non è valida, perché Claudio compare in entrambe le coppie. Per risolvere il problema
ragioniamo cercando di dare un ordine alle scelte e per cavalleria facciamo scegliere le ragazze.
Alice ha due alternative; fatta la scelta a Beatrice non resta che accontentarsi del ragazzo rimasto
libero. Le suddivisioni possibili sono quindi 2 ⋅ 1 = 2 (precisamente {( A, C );( B, D)} e
{( A, D);( B, C )} ).
d) Ragioniamo come sopra: la prima coppia di individui può essere scelta in
 4
 2
 
modi diversi. Fatta
la scelta il secondo gruppo è implicitamente definito (infatti volendo calcolare il numero di possibili
combinazioni rimaste otterremmo
 2
 2  = 1 ). In verità il calcolo non è ancora finito, perché abbiamo
 
contato troppi elementi, come mostreremo fra breve. Questo è accaduto perché per contare i casi
possibili abbiamo diviso il calcolo in “primo gruppo da due” e “secondo gruppo da due”, sebbene
le due coppie siano fra loro interscambiabili. Abbiamo quindi contato per esempio le suddivisioni
{( A, C );( B, D)} e {( B, D);( A, C )} separatamente, anche se essi rappresentino lo stesso caso. Il
 4
 2
risultato è quindi   = 3 (precisamente {( A, B);(C , D)} , {( A, C );( B, D)} e {( A, D);( B, C )} ).
2
Quest’ultimo caso è un esempio di combinazione di combinazioni.
23
Problema 2.13 - Cono con 2 gusti (soluzione a pagina 32)
Una gelateria offre gelati di 10 gusti differenti. Volendo prendere un cono con 2 gusti diversi, in quanti
modi si può scegliere?
Problema 2.14 – Picnic (soluzione a pagina 32)
Dieci escursionisti devono spartirsi 10 panini (un panino a testa). Sapendo che 5 panini sono al formaggio,
3 con il prosciutto e 2 con il tonno, in quanti modi diversi può avvenire la suddivisione del cibo?
Problema 2.15 – Alzare due dita (soluzione a pagina 33)
In quanti modi si possono tenere alzate due dita di una mano?
Problema 2.16 - Comitato equilibrato (soluzione a pagina 33)
In una classe di 20 alunni bisogna formare un comitato formato da 3 ragazzi e 3 ragazze. Sapendo che in
classe ci sono 7 maschi, in quanti modi si può effettuare la scelta?
Problema 2.17 – Essere Ittita (soluzione a pagina 33)
Usando l’alfabeto italiano, quante parole di 6 lettere esistono, tali che una lettera si ripete esattamente 3
volte e un’altra 2?
Siamo ormai arrivati quasi alla fine dei raggruppamenti classici, non manca che il seguente:
• Le combinazioni con ripetizione
Iniziamo subito con due esempi che, come vedremo, sono strettamente correlati. Darò in un
secondo momento la definizione di “combinazione con ripetizione”:
Problema Introduttivo 1
Almeno una biglia per contenitore
11 biglie identiche fra loro vanno disposte in 4 contenitori
A,B,C,D facendo in modo che nessun contenitore resti
vuoto. In quanti modi è possibile distribuire le biglie? (a
fianco → è mostrata una possibile ripartizione)
A
B
C
D
A
B
C
D
Problema Introduttivo 2
Biglie e contenitori senza restrizione
7 biglie identiche fra loro vanno disposte in 4 contenitori
A,B,C,D (volendo si potrebbero anche mettere tutte in
A). In quanti modi è possibile distribuire le biglie? (a
fianco → è mostrata una possibile ripartizione)
Affrontiamo il
24
Problema Introduttivo 1 e, trovata la soluzione, trattiamo il secondo caso.
Soluzione al
Problema Introduttivo 1: Almeno una biglia per contenitore
Mettiamo le 11 biglie in fila, come mostrato in basso.
Per scegliere quante biglie mettere in A, quante in B, in C e in D, possiamo utilizzare 3
bastoncini da frapporre fra le sferette a creare 4 gruppi. In basso è mostrato come esempio la
seguente distribuzione: 5 biglie in A, 2 in B, 1 in C e 3 in D.
Evidentemente qualsiasi posizionamento dei tre bastoncini rappresenta in modo univoco una
diversa ripartizione e, viceversa, ogni possibile ripartizione può essere rappresentata tramite i
3 bastoncini. Il problema può quindi essere ripensato come un posizionamento di 3
“interruzioni” tra le biglie. Con 11 biglie esistono 10 interstizi, per cui il nuovo problema èM
Problema Introduttivo 1 bis – Mettere 3 bastoncini
In quanti modi si possono scegliere 3 diverse posizioni su 10 possibili?
Soluzione 1 bis
La sequenza bastoncino Sì / bastoncino No può essere rappresentata dal disegno in basso
o più comodamente dalla “parola” xxxxBxBBxx. Ad ogni anagramma corrisponde un diverso
posizionamento dei bastocini e quindi una diversa ripartizione di biglie, per cui, ragionando in
termini di permutazioni con ripetizione, la soluzione ai problemi 1 e 1bis è
10!
. Più
3! ⋅ 7!
elegantemente, il problema di scegliere 3 caselle su 10 può essere interpretato come
10 
combinazione semplice, fornendo la soluzione (equivalente alla precedente)   .
3
E’ utile generalizzare quanto visto al caso di n biglie con k contenitori a disposizione.
Procedendo come sopra dovremmo mettere k − 1 bastoncini nei n − 1 interstizi. La soluzione è
 n − 1
10 
quindi 
 (guarda caso con n = 11 e k = 4 si ottiene proprio   )
3
 k − 1
Possiamo formalizzare quanto visto:
25
Osservazione
Il numero di modi per disporre n biglie in k contenitori, in modo che ciascun contenitore
 n − 1
.
 k − 1
contenga almeno una biglia, è 
Passiamo ora alla soluzione del problema 2, nel quale, per evitare ambiguità, indicherò il
numero di biglie con N ( n continuerà a essere il numero delle biglie del problema di tipo 1).
Soluzione al
Problema Introduttivo 2 - Biglie e contenitori senza restrizione
 n − 1

 k − 1
Invece di iniziare daccapo, partiamo dal problema 1 appena risolto. Ciascuna delle 
soluzioni rappresentava una distribuzione di 11 biglie nelle quattro scatole A,B,C,D, con la
condizione che in ogni scatola vi fosse almeno una biglia. Prendiamo una qualsiasi di queste
configurazioni e togliamo una biglia per scatola (vedi l’esempio in basso).
A
B
C
D
A
B
C
D
Quella che si ottiene sarà ora una ripartizione compatibile con il problema 2: infatti adesso le
biglie sono 7 e non si può escludere che qualche scatola sia vuota. Si può anche ragionare in
maniera opposta, prendendo una distribuzione compatibilie con il problema 2 e aggiungere una
biglia per contenitore: le biglie tornano ad essere 11 con la certezza che nessuna scatola resterà
vuota.
Questo discorso dovrebbe convincerci che i problemi 1 e 2 descrivono sostanzialmente la
stessa questione e hanno necessariamente la stessa soluzione. Indicati quindi con n il numero
di biglie di un problema di tipo 1 (restrizione di 1 biglia per scatola), con N il numero di biglie di
un problema ad esso associato di tipo 2 (alcune scatole possono anche essere vuote) e con k il
numero di contenitori, si ha la relazione N = n − k (nell’esempio precedente 7 = 11 − 4 ) e quindi
n = N + k . Possiamo allora dire che la soluzione al problema di tipo 2 con N biglie e k
 n − 1   N + k − 1
=
 , che con semplici passaggi matematici, può essere riscritto
 k − 1  k − 1 
 N + k − 1
come 
.
N


contenitori è 
Abbandoniamo ora per motivi estetici la lettera maiuscola N e tornando alle classiche lettere
k, n , possiamo riscrivere l’ultimo risultato così:
Osservazione
 k + n − 1
.
 n 
Il numero di modi per disporre n biglie in k contenitori è 
Nota bene: per tenere a memoria il senso delle singole lettere, potete immaginare che k
rappresenti il numero di kontenitori.
26
Definizione
Si chiama combinazione con ripetizione di n elementi di un insieme I composto da k oggetti
distinti, un gruppo di n elementi di I non necessariamente distinti e disposto in un ordine
qualsiasi.
Per capire bene la definizione, il nome “Combinazione con ripetizione” e la relazione che questa
definizione ha con i casi risolti precedentemente, consideriamo i seguenti problemi:
Problema Introduttivo 3
Tre biglie dall’urna senza restituzione
Da un’urna contenente 21 biglie etichettate con le lettere dell’alfabeto italiano, si estraggono
3 biglie mettendole in fila sul tavolo e ridisponendole infine in ordine alfabetico. Quante file di
questo tipo possono essere create?
Soluzione
Il riordino finale assicura l’irrilevanza dell’ordine di estrazione. Si tratta banalmente di
 21
 = 1330 modi.
3
scegliere 3 oggetti su 21 (combinazione semplice) e questo si può fare in 
Problema Introduttivo 4
Tre biglie dall’urna con restituzione
Un’urna contiene 21 biglie etichettate con le lettere dell’alfabeto italiano. Per tre volte di fila, si
estrae una biglia, si segna la lettera su un foglio e si rimette la biglia nell’urna. Alla fine si
comunicano le 3 lettere, leggendole in ordine alfabetico. Quante triplette del genere possono
essere create?
Soluzione
La differenza rispetto a prima è che la stessa biglia può essere estratta più di una volta: non
è una differenza da poco! Possiamo immaginare il problema in termini di contenitori da
riempire (come nel problema 2: Biglie e contenitori senza restrizione): le 21 scatole
rappresentano le lettere dell’alfabeto e ogni qualvolta una certa lettera viene estratta si pone
una biglia nel vano corrispondente. L’estrazione “F,A,F” verrebbe così rappresentata dalla
ripartizione con 2 biglie in F, una biglia in A e tutte le altre scatole vuote.
Abbiamo già risolto questo tipo di problema: ponendo k = 21 (numero di kontenitori, cioè di
lettere possibili) e n = 3 (numero di biglie, cioè di estrazioni) si ha la soluzione
 21 + 3 − 1   23 

 =   = 1771 .
3

 3
Il problema appena risolto rende giustizia al nome “Combinazioni con ripetizioni” e spiega il
legame con il problema dei contenitori.
27
Teorema sulle combinazioni con ripetizione
Il numero di combinazioni con ripetizione di k elementi di un insieme I composto da n
 k + n − 1
.
 n 
oggetti distinti è 
Esempio 2.18 - Una coppetta da 3 palline
Una gelateria offre gelati di 10 gusti diversi. Quante varianti di coppetta si possono comporre con 3
palline? Nota bene: è ammesso anche scegliere 3 volte lo stesso gusto.
Soluzione
Ragionando per analogia, i gusti sono i contenitori e le biglie rappresentano le palline di gelato. Si ha
quindi k = 10 , n = 3 e la soluzione
 10 + 3 − 1  12 

 =   = 220 .
3

 3
Esempio 2.19 - Ottenere 10
In quanti modi si può scrivere 10 come somma di 3 numeri naturali (0 escluso)? Bada bene, in questo
problema i “modi” 4+3+3 e 3+4+3 sono considerate combinazioni diverse.
Soluzione
Stavolta la soluzione si ottiene più facilmente pensando ai “bastoncini” piuttosto che alle formule.
Disponendo in fila 10 biglie e separandole con 2 bastoncini si possono creare tutte le somme desiderate
(vedi sotto la “ripartizione” 5+2+3.
Gli interstizi sono 9 , i bastoncini
9
2 , la soluzione non può che essere   = 36 .
 2
Esempio 2.20 - Pozioni e calcolo combinatorio
La leggenda narra che, gettando tutte in una volta 3 porzioni di spezia magica in un calderone traboccante
sangue di drago, si ottenga l’elisir del Bernoccolo Matematico. Quali spezie usare? Forse 2 porzioni di ali
di pipistrello e una di crine di unicorno, o magari 3 porzioni di capelli di fata, chi lo sa! Le spezie magiche
sono ben 50, chissà quante combinazioni esistono!
Soluzione
Le 3 spezie (che possono anche ripetersi, come si capisce dagli esempi), vanno gettate tutte insieme,
ragion per cui l’ordine della scelta è irrilevante. Possiamo come al solito immaginare i kontenitori delle
spezie ( k = 50 ) e il numero di biglie (numero di porzioni, n = 3 ). Si ha
28
 k + n − 1   52 
 n  =  3  = 22.100 .

  
Esempio 2.21 - Numeri in salita
Escludendo sempre la cifra 0, quanti numeri di 5 cifre esistono, tali che le loro cifre siano disposte in
ordine “uguale o crescente”? (“uguale o crescente” significa che una cifra può ripetersi, ma non può mai
essere seguita da una minore. Casi accettabili sono quindi 23.678 , 35.558 o addirittura 77.777 mentre
35.823 o 12.324 non andrebbero bene).
Soluzione
Il problema è complesso e per essere risolto necessita di un po’ di fantasia. Immaginiamo di scegliere 5
cifre qualsiasi da 1 a 9 (le cifre possono anche ripetersi) e di metterle in fila in ordine crescente. Si
ottengono così proprio i numeri “in salita” descritti dal problema. L’estrazione con restituzione da un’urna
contenente 9 cifre e la conclusiva “messa in ordine” (la quale rende irrilevante l’ordine di estrazione),
mostra che il problema è assimilabile al Problema Introduttivo 4 (pag. 27), cioè a una combinazione con
ripetizione. La soluzione è
 k + n − 1   9 + 5 − 1   13 
 n  =  5  =  5  = 1287

 
  
Osservazione
Ragionando in modo simmetrico, i numeri “in discesa” saranno anch’essi 1287 . I numeri “monocifra”
11.111 , 22.222 , M (9 in tutto) sono gli unici a far parte di entrambi i conteggi. Ricordando che i numeri
95 = 59.049 (disposizione con ripetizione), possiamo
5
concludere che i numeri “Sali e Scendi” sono i più frequenti ( 9 − 2 ⋅1287 + 9 = 56484 ), più del 95% del
composti da 5 cifre (senza mai usare lo 0) sono
totale.
Problema 2.18 – Elezione (soluzione a pagina 33)
In una classe di 20 alunni bisogna svolgere le votazioni per eleggere il rappresentante di classe, carica
per la quale si sono candidati in 4. Se ogni alunno può esprimere una sola preferenza e allo spoglio tutte
le schede risultano valide, in quanti modi diversi può concludersi la conta dei voti? Esempio: una “conta”
dei voti potrebbe essere la seguente: al candidato A 10 voti, al candidato B 0 voti, a C 5 voti e a D 5 voti.
Problema 2.19 - La confezione sorpresa (soluzione a pagina 33)
Un’azienda produce 3 tipi di vino e per l’anniversario della sua fondazione ha deciso di mettere in
commercio la confezione sorpresa: essa è composta da ben 7 bottiglie variegate e l’unica certezza è che
vi sia almeno una bottiglia per tipo. Quante confezioni sorpresa diverse posso esistere?
Problema 2.20 - Raggiungere l’aeroporto (soluzione a pagina 34)
9 amici devono raggiungere l’aeroporto e sono indecisi su come muoversi, se in macchina, con la navetta
o in treno. In quanti modi diversi possono effettuare la scelta? Vedi bene come interpretare la domanda:
a) Intendendo per “modi diversi” soltanto la distribuzione numerica delle persone. Così 3 persone in
macchina e 6 in treno rappresenterebbero un’unica configurazione, indipendentemente da chi
materialmente sale in macchina e chi in treno.
b) Intendendo con “modi diversi” non solo il numero, ma anche la composizione dei vari gruppi. Così
la configurazione per la quale Aldo, Bruno e Carla decidono di andare in macchina e tutti gli altri in
treno e la configurazione che vede andare in macchina Bruno, Davide ed Eleonora e tutti gli altri
in treno vanno conteggiate separatamente.
29
30
• Soluzioni dei problemi della Sezione 2
Soluzione 2.1 - Metal Detector
Si tratta di contare le permutazioni di una collezione composta da 5 elementi distinti. La soluzione è
5! = 120 .
Soluzione 2.2 - Una lettrice accanita
Indicati con A, B, C, D i titoli dei libri, bisogna mettere in un qualche ordine le 4 lettere, bisogna cioè
anagrammare la parola “ABCD”. Il numero di possibili anagrammi è 4! = 24 .
Soluzione 2.3 - Numeri e lettere
a) Per risolvere questo esercizio bisogna essere già abbastanza bravi. L’idea più semplice è quella
di anagrammare separatamente le 4 lettere e i 4 numeri per poi ricomporre “a pettine” gli otto
simboli. La soluzione è quindi
4! (anagrammi di ABCD) × 4! (anagrammi di 1234) , perché
ciascun anagramma letterale si lega a ciascun anagramma numerico (dando luogo a parole finali
diverse). Risulta quindi
4! × 4! = 24 ⋅ 24 = 576 .
b) Questa situazione è quasi identica alla precedente, consente però una scelta in più:
ricomponendo “a pettine” la parola finale si può iniziare da una lettere o da un numero. Visto che
precedentemente avevamo 576 scelte, la soluzione ora è 2 ⋅ 576 = 1152 .
Soluzione 2.4 - Anagramma con restrizione/1
I due esempi proposti (MARCO o CROMA) ci fanno capire che esistono due modi per affiancare la R e la
C. Concentriamoci inizialmente soltanto su “RC” e consideriamo la coppia come fosse un’unica lettera
inscindibile. Avremo così da anagrammare la nuova parola di 4 lettere distinte M A RC O . I modi per farlo
sono 4! = 24 . Stesso discorso per M A CR O , con la nuova “lettera” “CR”. Complessivamente gli
anagrammi permessi dal problema sono
4!+ 4! = 24 + 24 = 48 .
Soluzione 2.5 - Anagramma con restrizione/2
Le configurazioni che prevedono le due lettere separate sono più difficili da elencare di quelle che
prevedono le lettere appaiate. Dal problema precedente (che prevede una collezione con una struttura
equivalente a quella considerata qui), sappiamo che esistono 48 anagrammi di “OSTIA” con la S e la T
vicine. Del resto di “OSTIA” esistono complessivamente 5! = 120 anagrammi, per cui, operando la
sottrazione 120 − 48 , otteniamo proprio il numero che ci interessa. La soluzione è 120 − 48 = 72 .
Soluzione 2.6 - Anagrammi di Matematica
La parola “MATEMATICA” è composta da 20 lettere. La “M” e la “T” si ripetono ciascuna due volte, la “A”
tre. Applicando la formula si ottiene
10!
2! ⋅ 2!⋅ 3!
=
3.628.800
2 ⋅ 2⋅ 6
= 151.200 .
Soluzione 2.7 - Calcoli su Calcolo
Consideriamo la coppia CC come un’unica lettera. La nuova parola CC A L O L O ha 6 lettere: la L e la
O si ripetono due volte, per cui, applicando la formula, si ottiene
31
6!
2! ⋅ 2!
=
120
2⋅2
= 30
Soluzione 2.8 - Un piccolo robot
Affinché il robot, dopo 8 movimenti, si riporti al punto di partenza, è necessario che si muova 4 volte in
avanti e altrettante indietro, in un ordine qualsiasi (per esempio 3 volte avanti, 2 indietro, 1 avanti e 2
A
indietro). Indicando con le lettere
di 8 lettere
e I le due possibilità, si tratta di contare gli anagrammi della parola
8!
AAAA I I I I . Essi sono
4 volte
4! ⋅ 4!
4 volte
= 70 .
Soluzione 2.9 - Lucchetto a combinazione
Ogni rotella può trovarsi in 10 posizioni e ciascuna posizione di una singola rotella è compatibile con ogni
configurazione assunta dalle altre. Si tratta di una disposizione con ripetizione, il risultato è
104 = 10.000 .
Soluzione 2.10 - Targhe italiane
a) I “valori” assunti da ogni lettera e numero sono indipendenti gli uni dagli altri. Possiamo quindi
banalmente
moltiplicare
fra
loro
le
possibilità
per
ogni
posizione.
Il
risultato
è
22 × 22 ×10 ×10 ×10 × 22 × 22 = 22 ×10 = 234.256.000 .
4
3
b) Contiamo il numero delle targhe “mancanti”: le prime due lettere sono fisse su EE, gli altri simboli
possono assumere qualsiasi “valore”. Si tratta quindi di calcolare il numero di targhe del tipo
N N N L L . Esse sono 10 ×10 ×10 × 22 × 22 = 10 × 22 = 484.000 e quindi, la soluzione al
3
2
problema è 234.256.000 − 484.000 = 233.772.000 .
Soluzione 2.11 - Invito per 7
Ciascun invitato ha due opzioni: accettare l’invito o declinarlo. Scorrendo quindi le 7 persone possiamo
dire che esistono 2 × 2 × ... × 2 = 2 = 128 possibilità.
7
7 volte
Soluzione 2.12 - Coefficienti binomiali
 10  10 ⋅ 9 ⋅ 8 ⋅ 7
 4  = 1 ⋅ 2 ⋅ 3 ⋅ 4 = 10 ⋅ 3 ⋅ 7 = 210 ,
 
 12  12
 1  = 1 = 12 ,
 
7 7 ⋅6⋅5⋅4⋅3
 5  = 1 ⋅ 2 ⋅ 3 ⋅ 4 ⋅ 5 = 7 ⋅ 3 = 21 .
 
Soluzione 2.13 - Cono con 2 gusti
Si tratta di scegliere 2 elementi (in un ordine qualsiasi) da un insieme di 10 elementi distinti. È
evidentemente una combinazione semplice. La soluzione è
10 
 2  = 45 .
 
Soluzione 2.14 - Picnic
 10 
 5  modi diversi. Bisogna ora decidere a
 
 5
chi dare i 3 panini al prosciutto alle 5 persone rimaste. Questo può essere fatto in   modi. I due
 3
Distribuire i 5 panini al formaggio a 10 persone si può fare in
escursionisti rimanenti sono automaticamente destinatari dei panini al tonno . Riassumendo, la soluzione
al problema è
 10   5 
 5  ⋅  3  = 252 ⋅ 10 = 2520 .
   
32
Soluzione 2.15 - Alzare due dita
Stavolta si tratta di scegliere 2 elementi (in un ordine qualsiasi) da un insieme di 5 elementi distinti. La
soluzione è
 5
 2  = 10 .
 
Soluzione 2.16 - Comitato equilibrato
Una classe di 20 alunni con 7 maschi ha 13 femmine. La scelta della componente maschile può essere
7
 13 
fatta in   modi, quella della componente femminile in   . La soluzione finale è
3
3
 7   13 
 3  ⋅  3  = 35 ⋅ 286 = 10.010
   
Soluzione 2.17 - Essere Ittita
Prima di tutto scegliamo le lettere diverse fra loro: per designare quella che compare 3 volte esistono 21
modi, 20 per quella ripetuta e 19 per la solitaria. A questo punto tocca posizionarle: la lettera ripetuta tre
6
volte può essere disposta su sei posti liberi in   = 20 modi. Nelle 3 caselle ancora libere va ora
 3
posizionata la lettera solitaria (3 scelte) mentre gli ultimi due posti a disposizione vanno occupati con la
lettera (ripetuta) rimasta. Complessivamente le scelte sono 21 ⋅ 20 ⋅ 19 ⋅ 20 ⋅ 3 ⋅ 1 = 478.800 .
Soluzione 2.18 - Elezione
Ogni somma composta da 4 addendi naturali il cui risultato è 20, descrive uno spoglio diverso. Così per
esempio il caso A 10 voti, B 0 voti, C 5 voti e D 5 voti può essere rappresentato dalla somma 10+0+5+5.
Abbiamo già affrontato e risolto il problema di scrivere un numero come somma (Esempio 2.19 di pag. 28)
. La soluzione è
 k + n − 1   4 + 20 − 1   23 
 n  =  20  =  20  = 1771

 
  
Soluzione 2.19 - La confezione sorpresa
Ciascuna confezione può essere associata ad una somma di 3 numeri naturali maggiori di 0: così ad
esempio la somma 2+3+2 rappresenterebbe una confezione contenente 2 bottiglie del vino A, 3 bottiglie
del vino B e 2 del vino C. Questo tipo di problema è del tutto simile al
Problema Introduttivo 1 di pag. 24 e può essere assimilato a una combinazione con ripetizione mediante
la seguente riformulazione: Quante confezioni composte da 7 − 3 = 4 bottiglie esistono, senza che vi sia
la certezza che ogni tipo sia presente? Abbiamo ora una classica combinazione con ripetizioni con n = 4
e k = 3 . La soluzione è
 k + n − 1  3 + 4 − 1   6 
 n  =  4  =  4  = 15 .

 
  
33
Soluzione 2.20 - Raggiungere l’aeroporto
a) Mettiamo in fila i mezzi di trasporto (macchina, navetta, treno) e riscriviamo il numero 9 come
somma di 3 numeri naturali (0 incluso): possiamo così rappresentare tutte le suddivisioni possibili.
La somma 3+6+0 rappresenta ad esempio la configurazione “3 persone in macchina, 6 con la
navetta e nessuno in treno”, mentre 1+1+7 “1 persona in macchina, 1 con la navetta e 7 in treno”
e così via. Abbiamo già affrontato il problema di rappresentare un numero mediante somme
diverse (Esempio 2.19) e la soluzione è
 k + n − 1   3 + 9 − 1   11 
 n  =  9  =  9  = 55 .

 
  
b) Mettiamo in fila i 9 amici (Aldo, Bruno,M) e chiediamo ad ognuno come intende muoversi.
Ciascuno ha 3 possibili opzioni (macchina, navetta e treno): siamo di fronte a una disposizione
con ripetizione, la cui soluzione è 3 = 19.683 .
9
34
• Sezione 3 – Risolvere i problemi
È opportuno riassumere quanto visto finora e dare degli strumenti per poter affrontare i problemi
di Calcolo Combinatorio con ordine e metodo.
Per risolvere un problema si può operare sostanzialmente in due modi: il primo consiste nel
costruire, o anche solo immaginare, una struttura che contenga esattamente una volta tutte le
configurazioni ammesse dal problema stesso. In base alla regolarità della struttura si può
effettuare il calcolo dei casi possibili. Nel caso in cui si tratti di un albero a diramazioni
regolari, sarà necessario moltiplicare fra loro il numero di diramazioni di ciascun livello. La
struttura ad albero si presta molto bene a rappresentare i raggruppamenti che prevedano un
qualche tipo di ordine (prima scelta, seconda scelta e così via).
Il secondo modo di procedere è quello di riconoscere nel raggruppamento proposto una
configurazione nota (tra quelle sotto elencate) e applicare la rispettiva formula:
•
Permutazione semplice
•
Permutazione con ripetizione
•
Disposizione senza ripetizione
•
Disposizione con ripetizione
•
Combinazione semplice
•
Combinazione senza ripetizione
Come vi sarete accorti, la difficoltà non consiste tanto nell’applicare le formule, quanto nel capire
di volta in volta quale usare. Saper classificare i problemi è la capacità principale che bisogna
sviluppare nel Calcolo Combinatorio. Esistono vari stratagemmi per imparare quest’arte, il
primo dei quali si basa sul seguente schema. Esso si compone di due domande (ed
eventualmente di un quesito supplementare), che permettono di indirizzarvi verso il
raggruppamento corretto.
DOMANDA 1
La collezione con cui si ha a che fare, è
composta da oggetti tutti diversi fra loro?
Sì
Raggruppamenti
semplici, senza
ripetizione
No
Raggruppamenti
con ripetizione
Esempi
Lettere della parola UNICO, un mazzo di 40 carte, una classe
di studenti (elementi diversi fra loro)
Lettere della parola ENEIDE, un sacchetto di sassolini
identici, un’urna contenente due biglie dello stesso colore
(elementi ripetuti)
35
DOMANDA 3
Sì
DOMANDA 2
Gli oggetti del raggruppamento di partenza vanno
utillizati tutti nel raggruppamento finale?
Sì
Permutazioni
No
Disposizioni
Esempi
anagramma di una parola (tutte le
lettere della parola iniziale vanno
utilizzate)
Bisogna prendere in considerazione l’ordine con il
quale gli oggetti vengono
scelti/assegnati?
Eleggere un capo e un vice in un
gruppo (soltanto 2 persone vanno
elette)
Esempi
modi mettere in fila 8 persona
(ordine importante)
modi per scegliere 6 numeri da
giocare al SuperEnalotto (ordine non
importante)
No
Combinazioni
Purtroppo la semplicità dello schema proposto è spesso illusoria perché lascia spazio a
sottigliezze interpretative. Considerate ad esempio il caso di mettere n sassolini in k scatole
etichettate in maniera diversa, l’esempio che abbiamo usato per introdurre le combinazioni
senza ripetizione. Valutate poi la domanda 2: “Bisogna prendere in considerazione l’ordine con
il quale gli oggetti vengono assegnati?”. Bisogna rispondere “No” perché è irrilevante l’ordine
con il quale i sassolini vengono messi in una determinata scatola o “Sì”, perché le scatole non
possono essere mischiate fra loro e quindi il loro ordine è importante? Nel caso specifico è
corretta la risposta “No”, è però evidente come lo schema proposto possa non essere facile da
utilizzare.
Un metodo pratico è quello di ideare e tenere a memoria uno o più problemi – prototipo per
ciascun tipo di raggruppamento. Una volta che un situazione può essere schematizzata
mediante uno dei prototipi, essa è automaticamente classificata e quindi risolta. Consideriamo
ad esempio come problema prototipo quello di annerire k caselle su una fila composta da n
n
k 
caselle bianche. Esso è un esempio di combinazione, il cui numero complessivo è   . Il
problema di contare le diverse estrazioni del SuperEnalotto© (una pesca di 6 numeri su 90)
può essere riformulata immaginando una persona che annerisce i numeri estratti su una
schedina che contiene i numeri da 1 a 90: abbiamo così ridotto l’estrazione del
SuperEnalotto© al problema di annerire le caselle (di cui conosciamo la soluzione); le possibili
 90 
 = 622.614.630 .
6
estrazioni sono 
Si tratta quindi di ideare per ogni tipo di raggruppamento un problema possibilmente semplice,
chiaro e trasferibile a situazioni diverse: è un’operazione molto personale che riflette i gusti e la
mentalità di chi la fa. A seguire un possibile catalogo.
36
Tipo di raggruppamento
Problema prototipo
Permutazione senza ripetizioni
Contare gli anagrammi di
una parola composta da n
lettere diverse.
Soluzione
n!
Contare
i
modi
per
mischiare un mazzo di n
carte diverse.
Permutazione con ripetizioni
Contare gli anagrammi di
una parola composta da
lettere che si ripetono (per
esempio AAABBC).
n!
( k1 !)( k2 !) ...( kr !)
n0 × n1 × ... × nk
dove
Disposizioni senza ripetizioni
nj
è il numero di
scelte alla j − esima
assegnazione
Estrarre
k
biglie da
un’urna contenente
n
biglie diverse, mettendole
in
fila
nell’ordine
di
estrazione. Contare gli esiti
possibili.
37
Nel caso di disposizioni
semplici (per le quali
n j = n − j ), il risultato è
n!
( n − k )!
Tipo di raggruppamento
Problema prototipo
Soluzione
Disposizioni con ripetizioni
Da un’urna contenente n
biglie diverse, ripetere per k
volte la seguente operazione:
estrarre una biglia, segnare il
risultato, rimettere la biglia
nell’urna. Contare gli esiti
possibili.
nk
re sti tu zi o n e
Contare il numero di parole di
n lettere che si possono
formare utilizzando un alfabeto
composto da k simboli diversi.
Modi per annerire k caselle su
n.
Combinazioni semplici
Modi per estrarre un gruppo di
k persone da una classe di n
alunni.
Combinazioni con ripetizioni
A
B
C
D
Dati n sassolini e k scatole
etichettate, numero di modi
per disporre i sassolini.
Numeri di modi per scrivere un
numero intero n come somma
di k numeri naturali ordinati
(potendo utilizzare anche lo 0).
38
n
 
k 
 k + n − 1


 n 
• Problemi di riepilogo
Problema 3.1 - Impostare i canali (soluzione a pagina 39)
Considerando soltanto le 7 emittenti televisive più importanti, in quanti modi possibili si possono
sintonizzare i primi 7 canali della TV (in modo che su ogni canale vi sia esattamente una delle 7 reti)?
Problema 3.2 - Il popolo cariato (soluzione a pagina 48)
Per una strana malattia, in una nazione ogni cittadino ha almeno un dente cariato e nessun cittadino ha
esattamente gli stessi denti cariati di un altro. Ogni individuo è quindi univocamente identificato dalla
posizione delle sue carie. Quanti cittadini può avere al massimo questo Stato?
Problema 3.3 - Annerire le caselle (soluzione a pagina 48)
a) In quanti modi possono essere annerite k caselle in una tabella con n caselle bianche?
b) In quanti modi si possono annerire un numero arbitrario di caselle (eventualmente anche nessuna
o tutte) in una tabella con n caselle bianche?
Problema 3.4 - Sviluppo binomiale (soluzione a pagina 48)
Sviluppando il termine
(1 + x ) n = (1 + x ) ⋅ (1 + x) ⋅ ... ⋅ (1 + x ) si ottiene un polinomio di grado n del tipo
n volte
a0 + a1 x + a2 x + ... + an −1 x
2
a) Trovare i valori di
n −1
an
+ an x , dove per ogni k ak ∈ ℤ .
e
n
an−1 .
b) Trovare l’espressione generale del termine
ak
con 0 ≤ k ≤ n .
Problema 3.5 - Percorsi P → A (soluzione a pagina 49)
Il reticolo 8 × 8 disegnato a fianco rappresenta un insieme
di strade. Per arrivare da P ad A bisogna procedere
tassativamente verso destra, scegliendo ad ogni incrocio
una delle due biforcazioni (sempre che non si sia già arrivati
al bordo destro, nel qual caso la scelta è obbligata). Quanti
percorsi P → A esistono? (nel disegno a fianco è mostrato
un possibile itinerario → )
39
Problema 3.6 - Percorsi P → Q → A (soluzione a pagina 49)
In quanti modi è possibile partire da P e arrivare ad A ,
percorrendo 16 segmenti del reticolo 8 × 8 raffigurato a fianco e
passando tassativamente per
Q?
(da notare che percorrendo
16 segmenti si effettua un “percorso minimo”, si tratta infatti di
andare complessivamente di 8 segmenti a destra e di 8 in
basso).
Problema 3.7 - 4 pedoni bianchi e 4 pedoni neri (soluzione a pagina 49)
Avendo a disposizione 4 pedoni bianchi e 4 pedoni neri quanti modi esistono per disporli a casaccio su
una scacchiera (che ha 64 caselle numerate)?
Problema 3.8 - Numeri di 9 cifre (soluzione a pagina 50)
a) Quanti numeri di 9 cifre esistono (nel sistema decimale)?
b) Quanti di questi numeri sono composti da 9 cifre diverse?
Problema 3.9 - I numeri con il 3 (soluzione a pagina 50)
Considerando tutti i numeri fra 1 e 9999M
a) Quanti numeri contengono almeno una volta la cifra 3 ?
b) Quanti numeri contengono esattamente due volte la cifra 3 ?
Problema 3.10 - Sette con 3 dadi (soluzione a pagina 50)
In quanti modi diversi si può fare 7 lanciando 3 dadi?
Problema 3.11 - 22 giocatori (soluzione a pagina 50)
22 ragazzi vogliono dividersi in due squadre per disputare una partita. In quanti modi si può realizzare la
suddivisione in due gruppi (ovviamente A contro B equivale a B contro A)?
Problema 3.12 - Torneo di Tennis con 4 giocatori (soluzione a pagina 40)
4 giocatori di tennis vogliono organizzare un torneo di doppio (cioè con partite con 2 giocatori per parte) in
modo tale che ogni giocatore giochi almeno una volta contro ogni altro. Quante partite devono essere
disputate?
Problema 3.13 - Torneo di Tennis con 7 giocatori (soluzione a pagina 51)
7 giocatori di tennis vogliono organizzare un torneo di doppio in modo tale che ogni giocatore giochi
almeno una volta contro ogni altro. Quante partite devono essere disputate?
Problema 3.14 - Suddivisione in 4, 4 e 5 (soluzione a pagina 51)
Un classe di 13 alunni si divide in 3 gruppi di cui due da 4 e uno da 5 studenti. In quanti modi è possibile
effettuare la suddivisione seM?
a) I gruppi hanno mansioni diverse
b) I gruppi hanno tutti lo stesso compito
40
Problema 3.15 - Schedine del totocalcio con una colonna (soluzione a pagina 51)
Una schedina del totocalcio contiene una lista di 13 partite, ognuna delle quali può concludersi in tre modi
possibili ( 1, X ,2 ).
a) Quante schedine diverse esistono (considerando solo le schedine con un segno a partita)?
b) Quante di queste schedine contengono esattamente 8 1 , 3 X e 2 2 ?
c) Quante schedine contengono 8 segni di un tipo, 3 di un altro e 2 di un altro ancora?
Problema 3.16 - Parole con una ripetizione (soluzione a pagina 52)
Quante parole da 5 lettere (dell’alfabeto italiano) si possono creare considerando ammissibili soltanto le
parole formate da esattamente 4 lettere diverse (una quindi si deve ripetere)?
Problema 3.17 - Parole con alcune ripetizioni (soluzione a pagina 52)
Quante parole da 6 lettere (dell’alfabeto italiano) si possono formare considerando ammissibili soltanto le
parole formate da esattamente 3 lettere diverse?
Problema 3.18 - Estrazioni particolari (soluzione a pagina 53)
Un’estrazione del Lotto consiste in un pesca di 5 numeri da 1 a 90, nella quale l’ordine di estrazione è
irrilevante (per esempio le pesche 5,4,1,88,79 e 1,4,5,79,88 rappresentano la stessa estrazione ed è per
questo motivo che le estrazioni vengono normalmente citate in ordine crescente, cioè dal numero più
piccolo al più grande).
a) Quante diverse estrazioni prevedono 3 numeri sotto il 50 e 2 numeri dal 50 in su?
b) Quante estrazioni prevedono 5 numeri composti da 2 cifre e minori di 90 (quindi dal 10 al 89
inclusi) appartenenti a decine diverse? (come 32, 41, 61, 73, 89 mentre 13, 34, 35, 69, 79 non
andrebbe bene perché 34 e 35 appartengono entrambi alla decina del 30)
Problema 3.19 - Anagramma di se stesso (soluzione a pagina 41)
Quanti anagrammi della parola “ANAGRAMMA” esistono?
Problema 3.20 - Sassolini crescenti (soluzione a pagina 53)
Bisogna disporre 20 sassolini identici fra loro in 4 scatole contrassegnate con le lettere A,B,C,D, in modo
che nella scatola A vi sia almeno un sassolino, in B almeno 2, in C almeno 3 e in D almeno 4. In quanti
modi è possibile farlo?
Problema 3.21 - Somma di numeri pari (soluzione a pagina 41)
In quanti modi diversi si può rappresentare il numero 10 come somma di 3 numeri pari? Nota bene: in
questo esercizio lo 0 va considerato un numero pari. Rappresentazioni come 0+2+8 e 2+8+0 sono
considerate diverse (e concorrono entrambe al conteggio).
Problema 3.22 - La stanza arcobaleno (soluzione a pagina 54)
Le quattro pareti di una stanza vanno dipinte tutte di colori diversi. Avendo a disposizione 7 colori
differenti, in quanti modi è possibile pittare la stanza?
41
Problema 3.23 - Due taxi per nove (soluzione a pagina 42)
9 amici chiamano due taxi per andare in aeroporto. Il taxi A può trasportare 5 persone, il taxi B soltanto 4.
Quanti modi esistono per dividersi fra i due mezzi?
Problema 3.24 - Numeri disciplinati (soluzione a pagina 54)
Quanti sono i numeri naturali tali che ciascuna delle cifre “1”, “2”, M,”8”, “9” (quindi senza lo “0”) compaia
esattamente una volta (come ad esempio 952.137.648)?
Problema 3.25 - Un comitato studentesco (soluzione a pagina 54)
Un certo Liceo ospita 3 sezioni A,B,C per un totale di 15 classi e ciascuna classe è composta da 20
alunni. In quanti modi è possibile formare un comitato di 15 studenti in modo che ciascuna classe sia
rappresentata? Riporta il risultato in forma “semicalcolata” come ad esempio 20! × 15 o 3 − 20 .
15
Problema 3.26 - Zii ma non solo (soluzione a pagina 54)
Conta il numero di terne ordinate formate da 3 lettere (dell’alfabeto italiano) nelle quali una lettera
compare 2 volte (come A A B ).
Problema 3.27 - Biglie straetichettate (soluzione a pagina 55)
Un’urna contiene 9 biglie di tre colori diversi, contrassegnate da tre lettere e
da tre numeri diversi in modo che due biglie differenti abbiano al massimo
un carattere in comune (il colore, la lettera o il numero). Per chiarezza, a
fianco sono rappresentate tutte e nove le sferette.
Immagina di estrarre una biglia alla volta, disponendole in fila sul tavolo.
Quante estrazioni diverse esistono, tali che le tre biglie abbiano tutte e tre
un carattere in comune (cioè tali che le biglie abbiano tutte lo stesso colore,
la stessa lettera o lo stesso numero)?
Problema 3.28 - Delitto in scena (soluzione a pagina 55)
Una piccola compagnia formata da otto attori decide di inscenare un giallo. Bisogna assegnare i ruoli e il
copione prevede 1 ispettore, 3 poliziotti, 1 capobanda e 3 scassinatori. In quanti modi può essere
effettuata la scelta?
Problema 3.29 - I libri sullo scaffale (soluzione a pagina 42)
In una libreria ci sono 10 libri d’arte, 7 libri di biologia e 6 libri ci chimica. Volendo allinearli su un unico
scaffale in modo che i libri di una stessa materia siano tutti vicini fra loro, in quanti modi possono essere
disposti i 23 libri?
Problema 3.30 - Dodici biglie colorate e numerate (soluzione a pagina 55)
In un sacchetto stanno 4 biglie rosse numerate da 1 a 4, quattro
biglie blu numerate allo stesso modo e quattro biglie grigie,
sempre dello stesso tipo (vedi figura). Estraendo una biglia per
volta (per tre volte) e mettendole in fila sul tavolo, quante “file”
esistono per cuiM
a) le biglie siano tutte di colore e numero diverso
b) le biglie siano di colore uguale
42
2 1 1
2
1
3 2
4
3
3 4
4
c) le biglie siano di colore diverso
d) Supponendo di mettere le biglie poste sul tavolo in un altro sacchetto, quanti “sacchetti” diversi
esistono per gli esercizi a), b) e c)?
Problema 3.31 - I gironi agli Europei (soluzione a pagina 56)
I gironi degli europei di calcio prevedono 4 gruppi da 4 squadre. Ad ognuna delle sedici squadre è
assegnata a priori una fascia (le squadre della prima fascia sono dette teste di serie) e nessun gruppo può
contenere più squadre delle stessa fascia. Inoltre ogni gruppo gioca in uno stadio diverso. In quanti modi
diversi si possono essere sorteggiate le squadre, considerandoM?
a) Mlo stadio un elemento discriminante
b) Mlo stadio di gioco non rilevante
Problema 3.32 - Scuole medie e superiori (soluzione a pagina 56)
Una rappresentanza di docenti è composta da 11 insegnanti delle scuole medie e 9 insegnanti delle
superiori. Bisogna eleggere il portavoce del gruppo e il suo vice, tenendo presente che le due cariche
devono essere esercitate da insegnanti provenienti da ordini scolastici diversi (se il rappresentante è delle
medie il vice deve essere delle superiori e viceversa). In quanti modi può essere effettuata la scelta?
Problema 3.33 - Parole con due lettere uguali (soluzione a pagina 56)
Considerando l’alfabeto italiano (21 lettere), quante parole di 5 lettere esistono, tali che una ed una sola
lettera si ripete esattamente due volte (come AAGTR) o (ENSER)?
• Problemi difficilissimi
Concludiamo questa unità sul Calcolo combinatorio con tre problemi dedicati ai solutori più abili. Avverto
che questi problemi sono veramente difficili (per chiunque) e non possono essere risolti se non dopo
lunga riflessione, magari in modi diversi da quelli proposti qui.
Problema 3.34 - Sequenze ripide (soluzione a pagina 56)
Chiamiamo “sequenza ripida” una successione numerica composta da numeri naturali crescenti e diversi
fra loro. Sono sequenze ripide 0,4,5,10 o 4,9,100, mentre non fanno parte della categoria 4,4,8,9 (numeri
uguali), né tantomeno 5,2,8 (sequenza non crescente). La domanda è: quante sono le sequenze ripide
composte da 5 elementi che iniziano con lo 0 e finiscono con il 30?
Problema 3.35 - Somma di numeri dispari (soluzione a pagina 57)
In quanti modi diversi si può ottenere 21 sommando fra loro 5 numeri dispari? Attenzione, somme
composte dagli stessi elementi elencati in ordine diverso (come 1+3+5+3+9 e 5+9+1+3+3) vanno
conteggiate separatamente.
43
Problema 3.36 - Campo di frecce (soluzione a pagina 57)
Nello schema virtualmente infinito illustrato a destra
( → ) , in
ogni punto convergono tre frecce entranti e tre uscenti.
Immagina
di
partire
da
un
certo
punto,
percorrere
complessivamente 9 frecce (rispettando sempre la direzione
indicata dalla freccia stessa) e finire esattamente al punto di
partenza. Quanti itinerari diversi hanno questa caratteristica?
44
• Test di autovalutazione
(soluzioni a pagina 58)
(eventualmente da stampare, 3 pagine)
1) 20 persone si incontrano ed ognuna stringe la mano (una volta) ad ogni altra. Qual è il
numero complessivo di strette di mano?
A
20
B
95
C
380
D
190
E
400
2) Tra tutti gli anagrammi della parola CUORE, quanti hanno le due consonanti affiancate
(come p.e. CRUOE o RCUOE)?
A
120
B
60
C
48
D
24
E
10
3) Tra tutti gli anagrammi della parola ITALIA, quanti hanno le due consonanti affiancate (come
p.e. ITLAIA o ILTAIA)?
A
60
B
180
C
360
D
15
E
240
4) In una compagnia di 10 ragazzi, per Carnevale 4 si vestono da cowboy. In quanti modi è
possibile scegliere questi 4?
A
105
B
840
C
151.200
D
5040
E
210
5) Per il tuo compleanno ricevi 7 libri. Puoi decidere di leggerli tutti, nessuno o solo alcuni.
Quante alternative hai?
A
7
B
5040
C
128
45
D
49
E
5913
6) Da un guardaroba di 7 magliette e 5 paia di pantaloni bisogna scegliere 3 magliette e
altrettante paia di pantaloni per andare in ferie. In quanti modi diversi si può effettuare tale
scelta?
A
12.600
B
350
C
175
D
924
E
6300
7) Quanti anagrammi della parola SCUOLA iniziano con la lettera A?
A
720
B
120
C
24
D
360
E
15
8) Quanti numeri di 4 cifre contengono almeno una volta la cifra 0?
A
5904
B
9000
C
3168
D
2439
E
4999
9) Quante possibilità ci sono di distribuire, un frutto a testa, 4 mele, 3 pesche e 2 albicocche a 9
bambini?
A
381.024
B
1260
C
362.880
D
210
E
24
10) In una stanza ci sono tre sedie numerate e 5 persone in piedi. In quanti modi possono
accomodarsi questi ultimi? Specifichiamo che 2 e soltanto 2 devono restare in piedi e che la
scelta della sedia è considerata rilevante.
A
24
B
64
C
120
D
10
E
60
11) Un gruppetto di 3 ragazzi organizza una votazione segreta per eleggere il rappresentate.
Se ciascuno esprime una sola preferenza e tutte le schede risultano valide, in quanti modi può
concludersi lo spoglio? Nota bene: la votazione è segreta, dallo spoglio non si può vedere chi
ha espresso quale voto.
A
6
B
36
C
9
46
D
24
E
10
12) Tra tutti gli anagrammi della parola STRANA, quanti hanno le due vocali separate (come
p.e. STRANA o NASATR)?
A
B
96
C
600
D
120
E
24
240
13) A scopone scientifico, chi mischia le carte deve distribuire tutte le 40 carte del mazzo ai 4
giocatori (10 carte ciascuno), iniziando dall’avversario alla sua sinistra. In quanti modi diversi
può iniziare una partita?
A
B
C
 40 


 10 
40! 40!
40! 40!
+
+ ... +
+
40! 39!
1!
0!
40!
4
D
E
 40  30  20 




 10  10  10 
1! × 2! × ... × 40!
14) Nel Tennis semplificato una partita finisce non appena uno dei concorrenti vince 2 set e un
set si conclude non appena uno dei giocatori vince 6 game (senza tie-break o altro, per cui un
set potrebbe anche finire 6-5). Nel riquadro è mostrato un possibile esito:
A
6 5 6
B
0 6 3
Quanti diversi tabelloni (come quello mostrato) descrivono una vittoria di A ?
A
B
216
C
4320
D
468
E
125
750
Risposte:
1
2
3
4
11
12
13
14
5
6
47
7
8
9
10
.
• Soluzioni dei problemi della Sezione 3
Soluzione 3.1 - Impostare i canali
Ovviamente si tratta di un problema di permutazioni. La risposta è 7! = 5040 .
Soluzione 3.2 - Il popolo cariato
Ricordiamo che ogni persona ha 32 denti. Iniziando a contare dal molare in alto a destra fino al molare in
basso a sinistra possiamo numerare ogni dente, il quale o è cariato o è sano. Il numero di possibili
232 = 4.294.967.296 . Ricordandoci che in questo Stato una di
combinazioni è quindi 2 ⋅ 2 ⋅ 2 ⋅ ... ⋅ 2 cioè
32 volte
queste combinazioni è inammissibile (quella che prevede tutti i denti sani) possiamo dire che la nazione
può avere fino a 4.294.967.295 abitanti, cioè più del doppio della Cina.
Soluzione 3.3 - Annerire le caselle
a) Il primo problema equivale a chiedersi in quanti modi posso scegliere k oggetti in un insieme di n
n
 k  . Questo problema, sebbene molto semplice, offre
 
n
un’interpretazione del coefficiente binomiale   molto utile.
k 
 n n
n
b) Possiamo ricondurci al problema precedente e rispondere   +   + ... +   . Un altro modo per
 0 1
n
elementi. Per definizione la riposta è
vedere la cosa è considerare ogni casella singolarmente. Visto che essa può essere soltanto bianca o
nera la risposta è
n n
n
2 ⋅ 2 ⋅ ... ⋅ 2 = 2n . In definitiva abbiamo scoperto che   +   + ... +   = 2 n .
0 1
n
nvolte
Soluzione 3.4 - Sviluppo binomiale
a) Il coefficiente
xn
an
è quello che moltiplica il monomio di grado maggiore
xn . Notiamo che per ottenere
dal prodotto (1 + x ) ⋅ (1 + x ) ⋅ ... ⋅ (1 + x ) bisogna scegliere in ogni fattore il termine x . Questo vuol dire
n volte
che esiste un unico monomio del tipo
Il coefficiente
an
xn
è quello che moltiplica
e quindi la risposta è 1.
xn−1 . Per ottenere xn−1
dal prodotto (1 + x ) ⋅ (1 + x ) ⋅ ... ⋅ (1 + x )
n volte
bisogna scegliere n − 1 volte x e una volta l’uno (p.e.
x ⋅ x ⋅ ... ⋅ x ⋅1⋅ x ).
n fattori
scegliere un "1" su n fattori è ovviamente n .
48
Il numero di modi diversi per
k
k
In generale il coefficiente collegato a x è uguale al numero di modi per ottenere x dal prodotto
(1 + x ) ⋅ (1 + x ) ⋅ ... ⋅ (1 + x ) che è uguale al numero di modi per scegliere in k fattori la x . Questo è
n volte
ovviamente uguale a
n
n
  e quindi ak =   .
k 
k 
Soluzione 3.5 - Percorsi P → A
Qualsiasi percorso P → A consiste di 8 biforcazioni “Su”
altrettante biforcazioni “Giù”
(ց )
rappresentato
una
da
(ր )
e
e può essere schematicamente
sequenza
G S G G S G G S S S S S G G G S
del
(la
tipo
sequenza
proposta corrisponde al percorso rappresentato a fianco). Si tratta
quindi di contare gli anagrammi della parola di 16 lettere composta
G e S , ripetuti ciascuno 8 volte.
dai simboli
Esistono
16!
8! × 8!
anagrammi del genere, la risposta è quindi
Soluzione 3.6 - Percorsi
16!
8! × 8!
= 12.870
P →Q → A
Ricalchiamo i ragionamenti fatti per il problema precedente.
Eliminando dal reticolo i segmenti che non possono essere percorsi,
si arriva allo schema a fianco
( →)
. Si tratta di effettuare due
calcoli separati, il numero di percorsi minimi P → Q (cioè gli
anagrammi
numero
di
della
“parola”
percorsi
D D D D D D G G G ) e il
minimi
Q→A
(gli
anagrammi
di
D D G G G G G ).
Si tratta evidentemente di moltiplicare insieme i due numeri, per cui la soluzione è
9!
⋅
7!
6! × 3! 2! × 5!
= 1764
Soluzione 3.7 - 4 pedoni bianchi e 4 pedoni neri
Iniziamo con i pedoni bianchi e osserviamo che ci sono
caselle
abbiamo
ancora
 60 
4
 
modi
per
 64 
 4  modi per distribuirli. Visto che restano 60
 
mettere
i
 64   60  64 ⋅ 63 ⋅ 62 ⋅ 61 ⋅ 60 ⋅ 59 ⋅ 58 ⋅ 57
= 309.831.575.760 .
 4 ⋅ 4  =
1 ⋅ 2 ⋅ 3 ⋅ 4 ⋅1 ⋅ 2 ⋅ 3 ⋅ 4
   
49
neri.
In
definitiva
la
soluzione
è
Soluzione 3.8 - Numeri di 9 cifre
a) Nessun numero può iniziare con lo 0. Questo vuol dire che esistono 9 ⋅10 ⋅10 ⋅ ... ⋅10 = 9 ⋅10 numeri di 9
8
8 volte
cifre (si poteva anche calcolare così 999.999.999 − 99.999.999 = 900.000.000 ).
b) Visto che lo 0 non può stare all’inizio del numero esistono 9 modi per scegliere la prima cifra. Per la
seconda cifra ci sono di nuovo 9 possibilità (il numero delle cifre rimanenti, stavolta compreso lo 0), poi 8
e così via fino a 2. In definitiva la risposta è 9 ⋅ 9 ⋅ 8 ⋅ 7 ⋅ 6 ⋅ 5 ⋅ 4 ⋅ 3 ⋅ 2 = 3.265.920 .
Soluzione 3.9 - I numeri con il 3
a) Conviene considerare il problema inverso: quanti numeri non contengono nemmeno una volta la cifra
3 ? Non è necessario supporre che un numero non inizi con la cifra 0 (basterà considerare p.e. il numero
104 −1 numeri fra 1 e 9999 (lo 0 è escluso dall’intervallo). I numeri
4
3 sono invece 9 −1 (di nuovo dobbiamo escludere lo 0000) . La soluzione
0039 come 39). Esistono ovviamente
che non contengono la cifra
è quindi
104 − 94 = 10000 − 6561 = 3439 .
b) Calcoliamo preliminarmente il numero di modi per scegliere le posizioni decimali (immaginando un
numero come una sequenza ordinata di cifre) dove inserire i 3 : tale numero è ovviamente
 4
 2  . Sia la
 
prima casella libera che la seconda possono essere occupate in 9 modi diversi. La soluzione è quindi
4⋅3
 4
 2  ⋅ 9 ⋅ 9 = 1 ⋅ 2 ⋅ 81 = 486 .
 
Soluzione 3.10 - Sette con 3 dadi
Consideriamo preliminarmente un problema più generale: contare i modi per ottenere 7 sommando
insieme 3 numeri naturali qualsiasi (utilizzando quindi anche lo 0 e numeri maggiori di 6). Questo si può
 3 + 7 − 1  9 
 7  =  7  = 36 modi diversi. Dobbiamo ora togliere i casi incompatibili con i dadi. Per

  
nostra fortuna essi sono solo 3: 7 + 0 + 0 , 0 + 7 + 0 , 0 + 0 + 7 . La soluzione è quindi 36 − 3 = 33 .
fare in
Soluzione 3.11 - 22 giocatori
 22 
 11  modi per scegliere 11 elementi in un insieme di 22 elementi. Fatta questa scelta esiste
 
 11 
solo un modo (   = 1 ) per determinare la squadra avversaria. Il numero di disposizioni prima squadra 11 
 22 
seconda squadra è quindi   . Visto che sono richieste le possibili combinazioni (l’ordine tra le due
 11 
squadre non è rilevante) bisogna dividere il risultato per 2! . Complessivamente abbiamo quindi
 22 
 11  22 ⋅ 21 ⋅ 20 ⋅19 ⋅18 ⋅17 ⋅16 ⋅15 ⋅14 ⋅13 ⋅12
 =
= 352.716 possibilità.
Esistono
2!
1 ⋅ 2 ⋅ 3 ⋅ 4 ⋅ 5 ⋅ 6 ⋅ 7 ⋅ 8 ⋅ 9 ⋅10 ⋅11 ⋅1 ⋅ 2
50
Soluzione 3.12 - Torneo di Tennis con 4 giocatori
 4
 2  modi per formare una squadra da 2 giocatori con 4 persone a disposizione. Stabilita una
 
 2
squadra esiste un unico modo (infatti   = 1 ) per formare la squadra avversaria. Da come abbiamo
 2
Esistono
impostato il problema abbiamo calcolato il numero di disposizioni. In questo modo abbiamo dato
importanza all’ordine, per cui abbiamo contato due volte la partita A, B e B, A. Si tratta quindi di dividere
 4
 2 4⋅ 3
 =
= 3.
il risultato per 2! = 2 e ottenere
2
1⋅ 2 ⋅ 2
Notiamo che in questo caso il problema poteva essere risolto in modo più veloce con una
schematizzazione grafica.
Soluzione 3.13 - Torneo di Tennis con 7 giocatori
7
 2  modi per formare una squadra da 2 giocatori con 7 persone a disposizione. Stabilita una
 
5
squadra esistono   possibili coppie avversarie. Di nuovo abbiamo dato importanza all’ordine
 2
Esistono
(distinguendo fra la prima squadra scelta e la seconda) e quindi bisogna dividere il risultato finale per
 7  5
 2 ⋅  2 7 ⋅ 6⋅ 5⋅ 4
= 7 ⋅ 3 ⋅ 5 = 105 .
2! = 2 . Otteniamo     =
2!
1⋅ 2 ⋅ 1⋅ 2 ⋅ 2
Soluzione 3.14 - Suddivisione in 4, 4 e 5
13 
9
5
 4  modi per scegliere il gruppo A ,  4  per scegliere il gruppo B e  5  = 1 per scegliere
 
 
 
il
gruppo
Le
disposizioni
totali
sono
quindi
C.
 13   9  13 ⋅ 12 ⋅ 11 ⋅ 10 ⋅ 9 ⋅ 8 ⋅ 7 ⋅ 6
 4  ⋅  4  = 1 ⋅ 2 ⋅ 3 ⋅ 4 ⋅ 1 ⋅ 2 ⋅ 3 ⋅ 4 = 13 ⋅ 11 ⋅ 10 ⋅ 9 ⋅ 7 = 90.090 .
   
a) Esistono
b) Visto che i gruppi da 4 non sono fra loro distinguibili (cioè
A, B, C
e
B, A, C
sottintendono la stessa
 13   9   5 
 4  ⋅  4  ⋅  5  90090
     =
= 45.045 .
scelta) bisogna correggere il risultato nel seguente modo:
2!
Soluzione 3.15 - Schedine del totocalcio con una colonna
a) Ovviamente la riposta è
313 =1.594.323.
51
2
b) Consideriamo prima di tutto il seguente problema: in quanti modi si possono distribuire 8
righe? Ovviamente si tratta di contare le combinazioni e quindi la risposta è
sulle 13
13 
 8  . Le caselle ancora libere
 
 5
 3  possibilità. Le due caselle ancora libere devono
 
 2
"2" (infatti non c’è più scelta perché   = 1 ) e quindi la risposta è
 2
sono 5 e quindi per segnare 3
contenere
"1"
un
"X "
ci sono ancora
13   5  13 ⋅12 ⋅11⋅10 ⋅ 9 ⋅ 8 ⋅ 7 ⋅ 6 ⋅ 5 ⋅ 4 ⋅ 3
 8  ⋅  3  = 1⋅ 2 ⋅ 3 ⋅ 4 ⋅ 5 ⋅ 6 ⋅ 7 ⋅ 8 ⋅1⋅ 2 ⋅ 3 = 12.870 .
  
c) Nell’ultimo calcolo abbiamo ovviamente distinto fra i segni
1, X e 2 . Prendiamo una schedina qualsiasi
di quelle affrontate nel problema precedente e scambiamo fra di loro i segni (p.e. tutte le al posto di tutte
le X scriviamo
1
e viceversa). Anche questa nuova schedina ha le caratteristiche richieste. Possiamo
quindi dire che la riposta è
quindi la riposta è
13   5 
 8  ⋅  3
  
moltiplicato per le possibili permutazioni di 1, X , 2 (cioè 3! = 6 ) e
13   5 
 8  ⋅  3  ⋅ 3! = 12870 ⋅ 6 = 77.220 .
   
Soluzione 3.16 - Parole con una ripetizione
Per semplicità consideriamo inizialmente le due lettere all’inizio della parola. Questo vuol dire che non ci
occupiamo in questo momento dell’ordine delle lettere: stiamo quindi considerando delle combinazioni.
All’apparenza esistono 21⋅1⋅ 20 ⋅19 ⋅18 = 21⋅ 20 ⋅19 ⋅18 parole del tipo AABCD . In verità così facendo
contiamo per esempio sia AABCD che AABDC , parole che a questo punto del calcolo devono essere
considerate equivalenti. Dobbiamo quindi dividere per il numero di permutazioni delle tre lettere “diverse”
e considerare il numero
anagrammi
5!
2!
=
possibili
5 ⋅ 4 ⋅ 3 ⋅ 2 ⋅1
1⋅ 2
21 ⋅ 20 ⋅ 19 ⋅ 18
6
(perché
= 21 ⋅ 20 ⋅19 ⋅ 3 . Dobbiamo ora moltiplicare questo numero per gli
adesso
consideriamo
l’ordine
delle
lettere)
che
sono
= 5 ⋅ 4 ⋅ 3 . La soluzione è quindi 21 ⋅ 20 ⋅ 19 ⋅ 3 ⋅ 5 ⋅ 4 ⋅ 3 = 2.872.800 .
Soluzione 3.17 - Parole con alcune ripetizioni
Il problema è simile al problema Parole con una ripetizione. Ora però ci sono tre diverse possibilità
illustrate dai seguenti esempi: AABBCC , AAABBC , AAAABC . Notiamo che nella nostra
schematizzazione abbiamo premesso sempre le ripetizioni più numerose. Ragionando come prima
risolviamo i vari casi.
•
AABBCC : Il numero delle disposizioni è 21 ⋅1 ⋅ 20 ⋅1 ⋅19 ⋅1 . Visto che in questo primo passaggio
non consideriamo l’ordine delle lettere all’interno delle parole stiamo parlando di combinazioni.
Questo vuol dire che dobbiamo considerare come identiche fra loro le disposizioni AACCBB ,
BBCCAA e così via. Questo implica che ogni elemento è stato contato 3! = 6 volte invece che
52
21 ⋅ 20 ⋅ 19
una sola. Dobbiamo quindi considerare il numero
6
. A questo punto (e solo adesso)
6!
possiamo considerare l’ordine e calcolare il numero di anagrammi:
.
•
AAABBC : Il numero delle combinazioni è
6!
.
3!⋅ 2!
21 ⋅1 ⋅1 ⋅ 20 ⋅1 ⋅19 e gli anagrammi possibili sono
•
AAAABC : Il numero delle disposizioni è
21 ⋅1 ⋅1 ⋅1 ⋅ 20 ⋅19 , ma le ultime due lettere sono
“intercambiabili”. Il numero delle combinazioni è quindi
possibili sono
In
2!⋅ 2!⋅ 2!
totale
6!
4!
21 ⋅ 20 ⋅ 19
2!
=
21 ⋅ 20 ⋅ 19
2
e gli anagrammi
.
ci
sono
1
6!
6! 1 6! 
21⋅ 20 ⋅19 ⋅  ⋅
+
+ ⋅ =
 6 2!⋅ 2!⋅ 2! 3!⋅ 2! 2 4! 
quindi
 1 6 ⋅ 5 ⋅ 4 ⋅ 3 ⋅ 2 ⋅1 6 ⋅ 5 ⋅ 4 ⋅ 3 ⋅ 2 ⋅1 1 6 ⋅ 5 ⋅ 4 ⋅ 3 ⋅ 2 ⋅1 
= 21 ⋅ 20 ⋅ 19 ⋅  ⋅
+
+ ⋅
 = 21 ⋅ 20 ⋅ 19 ⋅ ( 5 ⋅ 3 + 6 ⋅ 5 ⋅ 2 + 3 ⋅ 5 ) =
1 ⋅ 2 ⋅ 3 ⋅1 ⋅ 2
2
1⋅ 2 ⋅ 3⋅ 4 
 6 1 ⋅ 2 ⋅ 1 ⋅ 2 ⋅1 ⋅ 2
= 21 ⋅ 20 ⋅ 19 ⋅ 90 = 718.200 modi.
Soluzione 3.18 - Estrazioni particolari
a) Bisogna
scegliere
3
numeri
su
49
e
2
su
41.
Ciò
può
essere
fatto
in
 49   41 
 3  ⋅  2  = 18.424 ⋅ 820 = 15.107.680 modi.
   
b) Il problema è molto diverso dal precedente. Scegliamo prima di tutto cinque decine tra le otto
 8
 modi. Per ogni decina esistono evidentemente 10 numeri
 5
ammesse: ciò può essere fatto in 
che possono estratti. Avendo a che fare con 5 decine e ricordando il calcolo iniziale si ha la
soluzione
8 5
 5  ⋅ 10 = 5.600.000
 
Soluzione 3.19 - Anagramma di se stesso
Si tratta di una permutazione con ripetizione: la “A” compare 4 volte e la “M” 2. La soluzione è quindi
9!
= 7560 .
4! × 2!
Soluzione 3.20 - Sassolini crescenti
La ripartizione di n sassolini identici in k scatole diverse senza alcuna condizione supplementare è
tipicamente una combinazione con ripetizione. Cerchiamo quindi di trasformare il problema proposto in
uno equivalente, ma privo della condizione sul numero di sassolini.
53
Consideriamo una qualsiasi ripartizione “con condizione” (così come esposta nel problema). Togliendo 1
sassolino da A, 2 sassolini da B, 3 da C e 4 da C otteniamo un ripartizione “classica” di 10 sassolini in 4
scatole (10 perché abbiamo tolto 10 sassolini dai 20 iniziali). Viceversa, partendo da una ripartizione
“classica” di 10 sassolini in 4 scatole e aggiungendo un sassolino ad A, due a B, tre a C e quattro a D
otterremmo una configurazione compatibile con la condizione posta dal problema. Questo dimostra
l’equivalenza
dei
due
problemi.
Possiamo
quindi
dare
la
soluzione:
 k + n − 1   4 + 10 − 1   13 
 n  =  10  =  10  = 286 .

 
  
Soluzione 3.21 - Somma di numeri pari
È possibile ricondurre i problema al conteggio delle somme di 3 numeri naturali qualsiasi la cui somma
faccia 5. Moltiplicando infatti ciascun numero per 2, si ottengono proprio le somme di 3 numeri pari con
somma
10.
Possiamo
 k + n − 1
 3 + 5 − 1
n
5


=
 
quindi
calcolare
la
soluzione
(combinazione
con
ripetizione):
7
 =   = 21 .
  5
Soluzione 3.22 - La stanza arcobaleno
Evidentemente l’ordine dei muri è rilevante, per cui si può ragionare una parete alla volta. Per dipingere la
parete a Nord si hanno 7 scelte diverse, per l’Est restano 6 opzioni, poi 5 per il Sud e infine 4 per la parete
Ovest. Complessivamente esistono 7 × 6 × 5 × 4 = 840 scelte possibili.
Soluzione 3.23 - Due Taxi per nove
Basterà scegliere chi sale sul taxi B (gli altri saliranno su A). Si tratta quindi di “estrarre” 4 persone su 9 e
9
ciò può essere fatto in   = 126 modi.
4
Soluzione 3.24 - Numeri disciplinati
Bisogna semplicemente contare il numero di permutazioni di “1”, “2”, M,”8”, “9” : sono 9! = 362.880 .
Soluzione 3.25 - Un comitato studentesco
Ogni classe ha 20 modi per esprimere il proprio rappresentante ed esistono 15 classi diverse. La
soluzione è quindi semplicemente
2015 .
Soluzione 3.26 - Zii ma non solo
L’alfabeto italiano ha 21 lettere. Per scegliere la lettera “doppia” vi sono 21 possibilità, per scegliere la
lettera singola ne restano 20. Ciascuna di queste scelte dà luogo a
3!
2!
54
= 3 terne differenti (per esempio
con le lettere A A B si possono formare le “parole”
A A B , A B A , B A A . Complessivamente
esistono quindi 21 × 20 × 3 = 1260 opzioni diverse.
Soluzione 3.27 - Biglie straetichettate
La prima estrazione può essere effettuata evidentemente in 9 modi. Supponiamo per semplicità che sia
uscita la biglia A1 (rossa). Seguendo il “carattere colore”, si hanno 2 possibilità (B2, C3 nei due ordini di
estrazione), rispettando il “carattere lettera” di nuovo due (A2, A3 e viceversa) e così anche per volendo
pescare sempre lo stesso numero (B1, C1 e viceversa). Esistono quindi 6 combinazioni che prevedono la
biglia A1 come prima estratta. Il discorso può essere esteso a tutte e nove le biglie: la soluzione è quindi
9 × 6 = 54 .
Soluzione 3.28 - Delitto in scena
Esistono 8 modi per scegliere l’ispettore. Restano ora 7 possibilità per scegliere il capobanda. Dai restanti
6
6 bisogna essere scegliere 3 attori per impersonare i poliziotti. Questo può essere fatto in   = 20 modi
3
e automaticamente restano definiti anche i 3 scassinatori. Riassumendo il numero di scelte è
8 × 7 × 20 = 1120 .
Soluzione 3.29 - I libri sullo scaffale
Per brevità denotiamo l’insieme dei libri d’arte con la A , l’insieme dei libri di biologia con la B e l’insieme
dei libri di chimica con la C . La prima scelta che si deve fare è quella di stabilire l’ordine delle tre
categorie, cioè le permutazioni possibili. Il risultato è ovviamente 3! . Una volta fatta questa scelta
possiamo “mischiare” all’interno di ogni gruppo i libri a piacimento: i libri d’arte possono essere permutati
fra loro in 10! modi, quelli di biologi in 7! e quelli di chimica in 6! maniere distinte. Il risultato è il prodotto
dei numeri da noi trovati, cioè
3! × 10! × 7! × 6! = 6 × 3.628.800 × 5040 × 720 = 79.009.136.640.000 .
Soluzione 3.30 - 12 biglie colorate e numerate
a) Visto che le biglie estratte vengono messe in fila su un tavolo, l’ordine è rilevante. La prima biglia può
essere una biglia qualunque e quindi ci sono 12 possibilità. Ci sono ora nel sacchetto ancora 8 biglie di
colore diverso dalla prima, 2 delle quali hanno però lo stesso numero. Questo implica che nella seconda
estrazione ci sono 6 modi per soddisfare le condizioni poste del problema. Notiamo che per l’ultima
estrazione il colore è ormai determinato, mentre per il numero restano 2 alternative. La risposta è quindi
12 ⋅ 6 ⋅ 2 = 144 .
b) Questo problema è molto più semplice del precedente. Come al solito per la prima biglia ci sono 12
possibilità. Restano nel sacchetto 3 biglie dello stesso colore, per cui nelle 2 seguenti estrazioni esistono
rispettivamente 3 e 2 modi per soddisfare le condizioni poste. La risposta è quindi 12 ⋅ 3 ⋅ 2 = 72 .
c) La prima estrazione è di nuovo libera e permette 12 alternative. A questo punto abbiamo 8 possibilità
per la seconda scelta e, una volta estratte 2 biglie, 4 per la terza. La soluzione è quindi 12 ⋅ 8 ⋅ 4 = 384 .
6) Mettendo le biglie in un sacchetto viene distrutto l’ordine, per cui per esempio le triplette R 1 , V 4 , B 2
e B 2 , R 1 , V 4 (che negli esempi precedenti rappresentavano soluzioni diverse) finiscono per
coincidere. Evidentemente esistono 3! = 6 modi per mischiare fra loro 3 biglie e quindi, ogni soluzione nel
55
sacchetto rappresenta 6 soluzioni diverse sul tavolo. Per ottenere la soluzione non dobbiamo quindi far
altro che dividere i risultati precedenti per 6 ed ottenere rispettivamente:
144
6
= 24 ,
72
6
= 12 e
386
6
= 64
Soluzione 3.31 - I gironi agli Europei
a) Supponiamo che gli stadi siano a Vienna, Zurigo, Salisburgo e Ginevra. Durante il primo sorteggio le
teste di serie vengono assegnate ai diversi stadi e quindi ci sono 4 ⋅ 3 ⋅ 2 ⋅1 = 24 possibilità. A questo punto
vengono distribuite le squadre di seconda fascia e, di nuovo, ci sono 4 ⋅ 3 ⋅ 2 ⋅1 = 24 modi per collocare le
squadre nei vari gruppi. Lo stesso discorso si ripete ancora per due volte e quindi complessivamente ci
sono
244 = 331.776 sorteggi possibili distinti.
b) Molti tifosi sono interessati solamente alle squadre da affrontare per cui considerano i sorteggi che per
esempio assegnano il gruppo composto da Italia, Francia, Olanda e Romania rispettivamente a Vienna o
a Zurigo sostanzialmente equivalenti (sempre che anche gli altri gruppi siano composti dalle stesse 4
squadre). Secondo questo punto di vista esistono soltanto
331.776
4!
= 13.824 sorteggi distinti, perché
“mischiare” fra loro i 4 gruppi non cambia il tipo di sorteggio. Un altro modo per vedere la cosa è
considerare come sorteggio “vero” soltanto quello che assegna le squadre dalla seconda fascia in giù e
quindi calcolare
243 = 13.824 .
Soluzione 3.32 - Scuole medie e superiori
Si tratta di scegliere un insegnante delle medie (11 possibilità), uno delle superiori (9 scelte) e poi
decidere chi dei due farà il portavoce (2 possibilità). Complessivamente le opzioni sono 11 ⋅ 9 ⋅ 2 = 198 .
Soluzione 3.33 - Parole con due lettere uguali
5
2
Esistono 21 modi per scegliere la lettera che si deve ripetere e vi sono   = 10 posizioni che le due
lettere uguali possono occupare. Restano ora tre caselle libere: la prima può essere occupata in 20 modi,
la seconda in 19 e la terza in 18. Le opzioni complessive sono quindi 21 ⋅10 ⋅ 20 ⋅19 ⋅ 18 = 1.436.400 .
Soluzione 3.34 - Sequenze ripide
Concentriamoci sulle “sequenza ripide” composte da soli tre numeri a , b , c con a < b < c , a ≥ 1 e
c ≤ 29 . Aggiungendo uno “0” all’inizio e un “30” alla fine, avremo successioni della forma richiesta (per
esempio le triplette 6,7,29 o 1,7,20 o 10,11,12 generano rispettivamente 0,6,7,29,30 e 0,1,7,20,30 e
0,10,11,12,30).
Per trovare il numero di “sequenza ripide” a , b, c consideriamole nella forma a '+ 1 , b '+ 2 , c '+ 3 , dove
a ', b ', c ' sono delle successioni “non decrescenti” (le chiamerò “successioni associate”), cioè tali che
a ' ≤ b ' ≤ c ' , a ' ≥ 0 e c ' ≤ 26 (così ad esempio 6,7,29 può essere vista come 5 + 1 , 5 + 2 , 26 + 3 dove
la successione associata 5,5,26 è composta da numeri non decrescenti, 1,7,20 può essere riscritta come
0 + 1 , 5 + 2 , 17 + 3
con
successione
associata
0,5,17
e
10,11,12
si
può
vedere
come
9 + 1 , 9 + 2 , 9 + 3 con successione associata 9,9,9. Abbiamo già “contato” successioni fatte così (pag.
56
 3 + 26 − 1   28 
 26  =  26  = 378 .

  
Andando a ritroso sono 378 le successioni di tipo a '+ 1 , b '+ 2 , c '+ 3 , quindi sono 378 le successioni
a , b, c e sono sempre 378 le successioni 0, a , b , c , 30 .
29 - Esempio 2.21 -
Numeri in salita), il loro numero complessivo è
Soluzione 3.35 - Somma di numeri dispari
Prendiamo 5 numeri naturali qualsiasi a,b,c,d,e tali che la loro somma faccia 8. Abbiamo quindi
a+b+c+d +e =8
2a + 2b + 2c + 2d + 2e = 16
per
cui
e
infine
( 2a + 1) + ( 2b + 1) + ( 2c + 1) + ( 2d + 1) + ( 2e + 1) = 25 .
quindi una cinquina
Ad ogni cinquina a,b,c,d,e di somma 8 corrisponde
A,B,C,D,E di somma 25 composta da soli numeri dispari. Viceversa, data una
successione di 5 numeri dispari di somma 25 si trova (ragionando a ritroso) una cinquina di numeri
naturali di somma 8. I due problemi sono quindi del tutto equivalenti e la soluzione del quesito si può
ricondurre
 5 + 8 − 1


al
facile
conteggio
delle
possibili
cinquine
naturali
di
somma
8.
Esse
sono
 12 
 =   = 495 .
 8
8
Soluzione 3.36 - Campo di frecce
Per risolvere il problema conviene suddividere le
frecce in 3 tipi: a fianco ciascuno di essi è
caratterizzato da un colore diverso (blu, rosso,
giallo).
Per percorre nove frecce e tornare al
punto di partenza bisogna necessariamente
passare per tante frecce blu (che portano in
basso a destra), quante frecce rosse (in basso a
sinistra), quante frecce gialle (direzione alto).
Il problema si può quindi ricondurre al numero di anagrammi della parola B B B R R R G G G .
Essi sono
9!
3!3!3!
= 1680 .
57
• Soluzioni del test di autovalutazione
1
2
3
4
5
6
7
8
9
10
11
12
13
14
D
C
A
E
C
B
B
D
B
E
E
E
D
C
Test 1. Si tratta di contare le coppie (non ordinate) in un gruppo di 20 persone, cioè
 20  20 ⋅ 19
 2  = 1 ⋅ 2 = 190 .
 
(la riposta giusta è quindi la D)
Test 2. Contiamo prima gli anagrammi contenenti la coppia CR e poi quelli contenenti RC. Considerando
CR come una coppia non divisibile (praticamente come un’unica lettera) otteniamo che il numero degli
anagrammi è 4! = 24 . Lo stesso discorso vale per il secondo caso e quindi la risposta è 2 ⋅ 24 = 48 . (la
riposta giusta è quindi la C)
Test 3. Ragioniamo come prima e consideriamo gli anagrammi della parola I-TL-A-I-A. Visto che la A e la
I si ripetono ognuna 2 volte, gli anagrammi sono
5!
2!⋅ 2!
= 30 . Lo stesso discorso vale per LT e quindi la
risposta è 30 ⋅ 2 = 60 . (la riposta giusta è quindi la A)
Test 4. Si tratta banalmente di contare il numero di modi per scegliere 4 elementi in un gruppo di 10, cioè
 10  10 ⋅ 9 ⋅ 8 ⋅ 7
 4  = 1 ⋅ 2 ⋅ 3 ⋅ 4 = 210 . (la riposta giusta è quindi la E)
 
Test 5. Ogni libro può essere letto oppure non letto. Partendo dal libro 1 fino al libro 7 possiamo dire che
ci sono 2 ⋅ 2 ⋅ ... ⋅ 2 = 2 = 128 possibilità. (la riposta giusta è quindi la C)
7
7 volte
Test 6. Per scegliere 3 magliette su 7 ci sono
invece
7 7⋅6⋅5
 3  = 1 ⋅ 2 ⋅ 3 = 35 modi. La scelta dei pantaloni offre
 
5 5⋅ 4 ⋅3
 3  = 1 ⋅ 2 ⋅ 3 = 10 alternative. La soluzione è quindi 35 ⋅ 10 = 350 . (la riposta giusta è quindi la B)
 
Test 7. Tenendo ferma una lettera restano ovviamente 5! = 120 modi per “mischiare” le altre. (la riposta
giusta è quindi la B)
Test 8. Conviene prima rispondere al problema opposto: quanti numeri di 4 cifre non contengono lo 0?
Con questo vincolo per ogni posizione decimale ci sono 9 possibili cifre (da 1 a 9) e quindi la risposta è
9 = 6561 . I numeri con 4 cifre sono in tutto 9000 ( 9999 − 999 = 9000 ) e quindi la risposta è
9000 − 6561 = 2439 . (la riposta giusta è quindi la D)
4
58
Test 9. Per decidere a chi distribuire le mele, bisogna scegliere 4 bambini in un gruppo di 9.
 9  9 ⋅8⋅ 7 ⋅ 6
 4  = 1 ⋅ 2 ⋅ 3 ⋅ 4 = 126 modi diversi. Restano ora 5 bambini tra i
 
5 5⋅ 4 ⋅3
quali bisogna sceglierne tre a cui donare le pesche. Ci sono   =
= 10 alternative. I restanti 2
 3  1⋅ 2 ⋅ 3
bambini riceveranno le albicocche e quindi la soluzione è 126 ⋅10 = 1260 . (la riposta giusta è quindi la B)
Evidentemente ciò può essere fatto in
Test 10.
Per rispondere a questo quesito conviene associare ogni sedia ad una persona (anziché
ogni persona ad una sedia). La sedia numero 1 può “scegliere” fra 5 alternative, la sedia 2 fra 4 (perché a
questo punto una persona si è già seduta) e la sedia 3 fra 3. La soluzione è quindi 5 ⋅ 4 ⋅ 3 = 60 . (la riposta
giusta è la E)
Test 11.
fianco
Soluzione: 10 (E). Basta costruire l’albero mostrato a
( → ) che rappresenta tutte le possibilità. Un’altra tecnica (più
potente perché sarebbe efficace anche per numeri più grandi) è la
seguente: scrivendo “3” come somma di 3 numeri naturali, possiamo
rappresentare tutti i possibili esiti della votazione: ad esempio 0+2+1
corrisponderebbe a 0 voti per A, 2 voti per B e 1 voti per C. Abbiamo
così utilizzato il problema prototipo delle combinazioni con ripetizione
 k + n − 1  5 
 n  =  3  = 10

  
con n = 3 e k = 3 . La formula dà
Test 12.
Soluzione: 240 (E). Contiamo prima di tutto gli anagrammi nei quali le AA sono affiancate.
Considerando AA come una lettera singola si hanno 5! = 120 modi per anagrammare AA STN .
6!
Complessivamente esistono
= 360 anagrammi di STRANA. Sottratti i 120 che non ci interessano, ne
2!
restano 240.
 40  30  20 
   (D). Esistono 10 modi per pescare 10 carte su 40. Esse
 10  10  10 
Test 13.
Soluzione: 
 30 
 
 10 
possibilità e
tabelloni
possibili:
spettano al giocatore alla sinistra di chi mischia. Il compagno di squadra ha ora
l’avversario alla destra
Soluzione: 62 + 63 + 63 = 468 (C).
Test 14.
La
 20 
  . Le 10 che restano sono del mescolatore.
 10 
seguente
A
6
6
B
X
Y
;
serie
di
riquadri
A
X
6
6
B
6
Y
Z
;
mostra
A
6
Y
6
B
X
6
Z
tutti
i
(dove X , Y , Z sono numeri compresi fra
0 e 5 e assumono quindi ciascuno 6 valori possibili). Esistono 6 × 6 = 36 modi per concludere una partita
di “tipo
3
I ”, 6 = 216 per una partita di “tipo II ” e altrettanti per una partita di “tipo III ”.
Complessivamente il numero è 36+216+216=468.
59
60
• Sezione 4 - Introduzione al Calcolo delle probabilità
• Gli eventi e la probabilità
L’esito di un esperimento o di una successione di eventi può essere soggetto a variabili che non
si controllano e in certi casi che non si possono in alcun modo conoscere. Nonostante questi
limiti normalmente è possibile determinare se un certo evento è più probabile o più frequente
(nel caso di un esperimento ripetibile) di un altro. Per misurare questa “aspettativa” si associa ad
ogni possibile evento un numero da 0 a 1, detto probabilità dell’evento. La probabilità 0 è da
intendersi come impossibilità mentre la probabilità 1 è associata ad un evento certo. Spesso tali
numeri sono espressi in percentuale (per cui al posto di 1 si scrive 100%).
Esempio
Lanciando una moneta, qual è la probabilità cheM?
a) Mesca testa o croce
b) Mnon esca né testa, né croce
Escludendo eventi fisicamente possibili ma matematicamente non ammessi (come il caso che la
moneta resti ferma sul bordo o che vada persa) possiamo dire che l’evento descritto nel punto
a), che chiameremo Ea , ha probabilità 1, mentre l’evento descritto nel punto b), cioè Eb , ha
probabilità 0. In simboli si scrive P( Ea ) = 1 e P( Eb ) = 0 .
Nel caso del lancio di due monete, gli eventi da prendere in considerazione sono le coppie
ordinate (C, C) , (C, T ) , (T , C) e (T ,T ) . Raggruppiamo gli esiti possibili in un insieme, il
cosiddetto spazio campionario Ω. Visto che ognuno dei possibili esiti è un elemento, l’insieme
è Ω = {(C , C ), (C , T ), (T , C ), (T , T )} .
Chiediamoci ora come rappresentare l’evento relativo al caso che in due lanci si ottenga
(indipendentemente dall’ordine) una volta testa e una volta croce. Riprendendo l’insieme di
prima possiamo rappresentare l’evento da noi desiderato come il sottoinsieme E1 di Ω
composto dagli elementi (C, T ) e (T , C) , cioè l’insieme E1 = {(C , T ), (T , C )} (vedi figura in
basso). Notiamo che con questo tipo di interpretazione ogni evento è da considerarsi come un
insieme di esiti, cioè come un sottoinsieme di Ω .
61
Questo vuol dire l’evento “in due lanci esce sempre testa” è l’insieme E2 = {(T , T )} (e non
l’elemento (T ,T ) ), mentre l’evento “esce un esito qualsiasi” è Ω stesso. Seguendo questa linea
di pensiero la probabilità non è nient’altro che una funzione che associa ad ogni sottoinsieme di
Ω un numero da 0 a 1 e in particolare P(Ω) = 1 e P(∅) = 0 . Vedremo più avanti come questo
modo di vedere le cose ci permetterà di utilizzare nel calcolo delle probabilità i consueti simboli
insiemistici.
Cerchiamo ora associare un valore numerico ai vari eventi. Ritornando all’esempio delle due
monete lanciate, è ragionevole supporre che ogni esito abbia la stessa probabilità. Visto che la
probabilità totale è 1, è naturale associare ad ogni evento contenente un singolo esito (come
E2 = {(T , T )} ) il valore
1
. Seguendo una linea di pensiero analoga possiamo attribuire
4
all’evento “esce una volta testa e una volta croce, indipendentemente dall’ordine”
E1 = {(C , T ), (T , C )} ) il valore
(cioè
1
(infatti tale insieme contiene metà degli elementi di Ω ). Nel
2
disegno vediamo rappresentati in verde e in rosso i due eventi appena descritti.
Per calcolare la probabilità che in due lanci esca almeno una volta testa possiamo osservare
che tale evento è l’unione degli eventi E1 ed E2 sopra rappresentati. E’ naturale sommare le due
probabilità ed ottenere P ( E1 ∪ E2 ) = P ( E1 ) + P ( E2 ) =
1 1 3
+ = . Sebbene tale risultato sia
2 4 4
corretto bisogna prestare attenzione a non generalizzare il metodo usato. Il seguente caso
fornisce un esempio di un possibile “tranello”:
62
Sia E1 il solito evento descritto sopra ed E3 l’evento “nel secondo lancio esce testa”, quindi
E3 = {(C , T ), (T , T )} . E’ facile osservare che P ( E3 ) = P ( E1 ) =
1
. Evidentemente l’unione dei due
2
eventi corrisponde esattamente all’evento discusso in precedenza (“esce almeno una volta
testa”) ma stavolta non vale la regola P ( E1 ∪ E3 ) = P ( E1 ) + P ( E3 ) (infatti otterremmo
P ( E1 ∪ E3 ) = 1 , cioè la certezza assoluta). Il motivo è di facile comprensione: i due insiemi non
sono disgiunti e sommando le due probabilità si conta due volte la probabilità dell’evento
comune E1 ∩ E3 = {(C , T )} . Correggere la regola non presenta comunque grosse difficoltà: per
ogni E1 , E1 ⊆ Ω deve valere P ( E1 ∪ E2 ) = P ( E1 ) + P ( E2 ) − P ( E1 ∩ E2 ) . Nel caso E1 ∩ E2 = ∅
la regola resta comunque valida perché P ( ∅ ) = 0 .
L’ultima considerazione riguarda l’evento complementare ad E che denoteremo con E .
Evidentemente i due insiemi sono disgiunti e la loro unione è Ω . Applicando la regola appena
( )
esposta possiamo concludere che P E = P ( Ω ) − P ( E ) = 1 − P ( E ) . Un esempio di evento
complementare è rappresentato nel seguente disegno:
E = {(C , T ), (T , C ), (T , T )} e E = {( C , C )}
(Ovviamente per ogni insieme risulta E = E )
Riassumiamo quanto detto fino a questo punto:
•
Ω è l’insieme di tutti gli esiti possibili e ogni evento E è un sottoinsieme di Ω , cioè
E⊆Ω
•
P (Ω ) = 1 e P (∅ ) = 0
•
Per ogni E1 , E1 ⊆ Ω risulta P ( E1 ∪ E2 ) = P ( E1 ) + P ( E2 ) − P ( E1 ∩ E2 )
•
P E = 1− P ( E )
( )
63
• Metodi di calcolo
Vediamo ora alcuni esempi concreti che ci permetteranno di introdurre e di spiegare le
metodologie più semplici utilizzate nel calcolo delle probabilità
Esempio: 2 volte testa, 2 volte croce
Supponiamo di lanciare una moneta per 4 quattro volte di fila. Qual è la probabilità che esca 2
volte testa e 2 volte croce?
Soluzione 1
La semplicità del problema consente di utilizzare
svariate strategie risolutive. Possiamo per esempio
utilizzare la rappresentazione ad albero e trovare i casi
che soddisfano le condizioni poste del problema. Nella
figura a fianco vediamo un diagramma del genere: le
frecce sulla destra puntano sui casi favorevoli (nel
calcolo delle probabilità con “caso favorevole” si
intende un evento che soddisfa le condizioni poste). Il
numero dei casi possibili corrisponde al numero delle
foglie, in questo caso 16 . E’ ragionevole pensare che
la probabilità sia data dal rapporto fra gli esiti favorevoli
e gli esiti possibili e che quindi, alla luce del
diagramma, p ( E ) =
6 3
= .
16 8
Soluzione 2
Sebbene il metodo appena descritto si sia rivelato
efficace, è evidente che per un problema più complesso sarebbe troppo laborioso disegnare
tutto l’albero. Ritornando alla definizione di probabilità suggerita prima (il rapporto fra casi
favorevoli e casi possibili), possiamo semplicemente calcolare il numero dei casi utilizzando le
formule del calcolo combinatorio: gli esiti possibili sono evidentemente 2 = 16 perché ognuno
dei quattro lanci ha 2 esiti possibili. Per quanto riguarda i casi favorevoli osserviamo che il
problema è analogo al seguente: in quanti modi si possono scegliere 2 elementi (mettiamo i
4
 4
 2
lanci nei quali esce testa) su 4 (i lanci totali)? La risposta è evidentemente   = 6 e quindi
otteniamo il risultato di prima. Come vedremo fra poco, non sarà sempre possibile ragionare in
questo modo perché non è sempre vero che tutti gli esiti siano equiprobabili. Possiamo
comunque enunciare la seguente proposizione:
Se gli esiti possibili sono fra loro equiprobabili allora la probabilità che si verifichi
l’evento E è data dalla seguente formula: P( E ) =
64
numero di casi favorevoli
.
numero di casi possibili
Soluzione 3
Introduciamo ora un metodo che a prima vista sembrerà quasi identico al precedente ma che
concettualmente opera in maniera diversa. Attribuiamo ad ogni ramo dell’albero la probabilità
dell’esito ad esso associato: visto che nel nostro caso in ogni lancio i due esiti possibili sono
equiprobabili ogni ramo avrà probabilità p =
1
.
2
Per fissare le idee supponiamo di voler ora stabilire
la probabilità dell’evento E2 = {(C , T )} . Il nodo
relativo a tale evento è evidenziato nella figura con
un tondino arancione. Partiamo ora dalla radice
dell’albero e ripercorriamo i rami fino ad arrivare ad
E2 . La probabilità di trovarsi, dopo il primo lancio, sul
1
. A questo punto la probabilità di
2
1
e quindi
“arrivare a destinazione” è di nuovo
2
1 1
complessivamente
P ( E2 ) = ⋅ . In generale
2 2
ramo giusto è
possiamo dire che la probabilità di ogni nodo (ogni
nodo rappresenta un evento) è data dal prodotto
delle probabilità dei rami che lo collegano alla radice
dell’albero, nel nostro caso appunto
1 1 1
⋅ = .
2 2 4
Ragionando in questa maniera è facile accorgersi che i nodi finali hanno tutti la stessa
1 1 1 1 1
⋅ ⋅ ⋅ = . Visto che i nodi evidenziati sono in tutto 6 possiamo dire che la
2 2 2 2 16
1 3
probabilità dell’evento desiderato E è P ( E ) = 6 ⋅
= .
16 8
probabilità
La differenza con il metodo visto sopra è la seguente: mentre in precedenza abbiamo ottenuto la
probabilità dell’evento favorevole calcolando un rapporto fra numero di casi, stavolta abbiamo
sommato fra loro le probabilità di tutti i casi favorevoli. In altre parole abbiamo utilizzato la
seguente proprietà: se E1 ∩ E2 = ∅ allora P ( E1 ∪ E2 ) = P ( E1 ) + P ( E2 ) (avendo discusso di
eventi contenenti un solo esito sicuramente E1 ∩ E2 = ∅ ). Possiamo enunciare quanto visto
nella seguente proposizione:
La probabilità di un evento E composto da un certo numero di esiti è uguale alla
somma delle probabilità dei singoli esiti.
Ricordo che nel nostro modello ogni “esito” è una successione ordinata del tipo (T , C, C, T ) .
Vediamo come questo modo di ragionare permette di risolvere problemi più complessi
65
Esempio: Il pugile del 70%
Supponiamo che un pugile, indipendentemente dall’avversario e dalle partite giocate in
precedenza, abbia una probabilità di vittoria del 70%, scriviamo p =
7
. Il pugile in questione
10
partecipa ad un torneo nel quale sono previsti 5 incontri senza eliminazione diretta (cioè anche
perdendo l’atleta disputa comunque l’incontro successivo).
a) Con che probabilità il pugile vince tutti e cinque gli incontri?
b) Con che probabilità ne vince soltanto 3?
Soluzione: Il pugile del 70%
Costruiamo di nuovo l’albero delle probabilità prestando attenzione al fatto che i rami non sono
più equiprobabili: i rami associati ai tondini verdi (che corrispondono alle vittorie) hanno
probabilità p =
7
3
, mentre i rami “rossi” hanno probabilità 1 − p , cioè
. L’albero in figura in
10
10
verità non rappresenta esattamente il nostro problema (infatti gli incontri rappresentati sono solo
4) ma ci permetterà comunque di accompagnare i nostri ragionamenti con un spiegazione
grafica. L’unico tragitto che corrisponde al primo problema percorre tutti i rami in alto perché è
composto da 5 vittorie. La probabilità di questo evento è data dal prodotto delle probabilità dei
5
singoli casi e cioè
7 7 7 7 7  7 
⋅ ⋅ ⋅ ⋅ =   ≅ 0,168 . Il
10 10 10 10 10  10 
pugile vince tutti gli incontri con una probabilità di ca.17%.
Il secondo problema è più complesso. Possibili esiti
favorevoli sono per esempio le successioni S ,V , S ,V ,V o
V ,V , S, S ,V (la V sta per vittoria, la S per sconfitta).
Notiamo che ognuno di questi esiti percorre 3 volte un
ramo di probabilità
1− p =
probabilità
p=
7
e due volte un ramo di
10
3
, indipendentemente dall’ordine.
10
Questo vuol dire che gli esiti favorevoli hanno probabilità
 7 
 
 10 
3
 3 
⋅ 
 10 
2
e sono fra loro equiprobabili (mentre in
generale due esiti qualsiasi non lo sono). Non ci resta
allora che trovare il numero di tali casi. Ragionando come
 5 5⋅ 4
= 10
 2  1⋅ 2
nel caso precedente osserviamo che per “scegliere” quali partite perdere ci sono   =
 7 
modi. Siamo ora in grado di calcolare la probabilità richiesta che sarà 10 ⋅  
 10 
3
2
 3
⋅   ≅ 0, 309 .
 10 
Il pugile ha quindi una probabilità del 31% (ca.) di vincere esattamente 3 incontri.
Il problema da noi proposto si presta ad una immediata generalizzazione. Volendo calcolare le
probabilità associate a tutti gli eventi possibili potremmo utilizzare la seguente tabella:
66
Numero di incontri vincenti
4
5  7 
 ⋅ 
 0   10 
Probabilità
Numero di incontri vincenti
n
n
5
 3
⋅ 
 10 
0
 3
⋅ 
 10 
3
3
 5  7 
 ⋅ 
 1   10 
2
 5  7 
 ⋅ 
 3   10 
Probabilità
n
0
5
4
1
 3
⋅ 
 10 
1
2
n
1
3
 3
⋅ 
 10 
5
 3
⋅ 
 10 
5  7 
 ⋅ 
 2   10 
2
0
1
5  7 
 ⋅ 
 4   10 
 3
⋅ 
 10 
4
 5  7 
 ⋅ 
 5   10 
0
 n 
 valgono n ).
 n − 1
(Ricordiamo che   e   valgono 1 e che   e 
Notiamo che anche la soluzione da noi data al primo problema calcolato può essere espressa in
 5
 0
questa maniera: modi per ottenere 0 sconfitte in 5 partite   = 1 moltiplicato per le varie
5
 7 
probabilità di tutti i rami “vincenti percorsi”   moltiplicato per le varie probabilità di tutti i
 10 
0
 3
“rami perdenti” percorsi   . Siamo quindi in grado di dare una formula generale per il
 10 
problema del pugile del 70%:
k
5−k
5  7   3 
o anche
La probabilità di vincere esattamente k partite su 5 è   ⋅   ⋅  
 k   10   10 
5 k
5− k
7
rappresenta la probabilità di vittoria per ogni incontro.
  ⋅ p ⋅ (1 − p ) , dove p =
10
k
Notiamo che la formula può essere ulteriormente generalizzata anche per un numero di incontri
qualsiasi e per una probabilità di vittoria arbitraria. Basta rispettare la seguente linea di pensiero:
qualsiasi percorso che contenga k rami di probabilità di p e n − k rami di probabilità 1 − p ha
probabilità p (1 − p)
k
n −k
n
k 
. Del resto esistono   percorsi del genere e quindi la probabilità di
vincere k incontri su n sapendo di vincere ogni incontro con probabilità p è uguale a
n k
n−k
  ⋅ p ⋅ (1 − p ) .
k
L’ultima formula si può applicare a moltissimi problemi comuni è prende il nome di distribuzione
binomiale o distribuzione di Bernoulli ed esprime la probabilità dell’evento al variare del
parametro k .
67
• La distribuzione binomiale
Definizione
Si dice esperimento di Bernoulli una sequenza di n prove con le seguenti caratteristiche:
• Il risultato di ogni prova può essere soltanto un successo o un fallimento
• Il risultato di ciascuna prova è indipendente dai risultati delle prove precedenti
• La probabilità di successo p (e di conseguenza anche la probabilità di insuccesso
1− p ) è costante in ciascuna prova.
Possiamo anche esprimere quanto appena enunciato nel seguente modo: qualsiasi problema
sia schematizzabile da una albero delle probabilità come quello rosso-verde mostrato in
precedenza è un esperimento di Bernoulli.
Teorema
La probabilità che in un esperimento di Bernoulli composto da n prove (ciascuna delle quali
con probabilità di successo p ), si abbiano esattamente k successi, è data dalla formula
n
n −k
Pn (k ) =   ⋅ p k ⋅ (1 − p )
k 
Esempio 4.1
– Mele marce/1
Il 10% delle mele di un certo agricoltore sono marce, ma esse sono mischiate in modo
perfettamente casuale con quelle sane. Comprando una cassa da 10 mele, qual è la probabilità
che essa ne contenga k marce?
Soluzione
Immaginando di pescare una mela per volta possiamo rappresentare il nostro problema con
l’albero rosso-verde presentato precedentemente. Il problema è quindi un esperimento di
10 − k
k
10   1   9 
Bernoulli e la soluzione è P( Ek ) =   ⋅   ⋅  
, dove Ek rappresenta l’evento “ k mele
 k   10   10 
1
marce”, al posto di p abbiamo inserito
e al posto di n 10.
10
Cerchiamo ora di analizzare criticamente i risultati numerici da noi trovati. Premettiamo una
considerazione: il fatto che in media una mela su dieci sia marcia non implica che ogni cassa da
10 ne contenga una. E’ anzi straordinario che con la sola ipotesi che le mele marce siano
distribuite in modo casuale, siamo in grado di prevedere, in media, quante casse contengano
quante mele marce.
Applicando la formula a k = 0,1,2,...,10 otteniamo i seguenti risultati:
68
Numero di mele marce
per cassa (da 10)
0
1
2
3
4
5
6
7
8
9
Probabilità
0,348678
0,38742
0,19371
0,0573956
0,0111603
0,00148803
0,000137781
0,000008748
0,00000036
0,000000009
10
<
1
109
Come era immaginabile il risultato più comune è quello di trovare una mela marcia per cassa.
Ciononostante appare a prima vista sorprendente che tale probabilità sia “solo” del 39% e che la
probabilità di trovare soltanto mele sane sia così alta. Ancora più sorprendente è la distribuzione
relativa al seguente esempio:
Esempio 4.2
– Mele marce/2
Il solito agricoltore vende anche confezioni da 5 mele ciascuna. Come sono distribuite le
probabilità dei vari casi (nessuna mela marcia per confezione, una mela marcia, ecc.)?
Soluzione
Ovviamente il problema è del tutto analogo a quello precedente, l’unica differenza è che stavolta
n
k 
nella formula generale pn ( k ) =   ⋅ p k ⋅ (1 − p )
n− k
Vediamo le varie probabilità:
69
al posto di n inseriamo 5.
Numero di
mele marce
per confezione (da 5)
0
1
2
3
4
5
Probabilità
0,59049
0,32805
0,0729
0,0081
0,00045
0,00001
Come vediamo in questo caso la probabilità di pescare una mela marcia in una confezione è
solamente del 33% ca. Anche sommando fra loro tutte le probabilità legate a eventi “cattivi” (fino
al caso limite di una confezione senza nemmeno una mela sana), si raggiunge solo poco più del
40%.
A questo punto sorge la domanda: ma questa distribuzione di probabilità corrisponde veramente
all’ipotesi che le mele marce siano il 10% del totale? La risposta a questo quesito introduce un
nuovo importantissimo tema.
• Il valore atteso
Per rispondere all’ultima domanda che ci siamo posti supponiamo di comprare 1000 confezioni
per un totale di 5.000 mele. In media ca. 590 confezioni saranno sane (perché la probabilità di
trovare confezioni del genere è di 0.59049), ca. 328 confezioni conterranno 1 mela marcia e così
via. Sommiamo ora il numero di mele marce ottenute con questo calcolo.
Numero di
confezioni
590
328
73
8
1
0
Numero di mele marce
per confezione
0
1
2
3
4
5
TOTALE
70
Numero totale
di mele marce
0
328
146
24
4
0
502
Come vediamo il risultato è, a meno di errori di approssimazione, in linea con l’ipotesi che il 10%
delle mele fosse marcio. Un breve calcolo mostra che anche il risultato del problema precedente
corrisponde all’ipotesi.
Come già accennato, l’operazione da noi compiuta è di grande interesse. Per semplificare il
problema abbiamo supposto di comprare ben 1000 confezioni, ma questa ipotesi non era certo
necessaria: moltiplicando fra loro la probabilità p(Ek ) (probabilità di comprare una confezione
con k mele marce) e il numero di mele marce k avremmo ottenuto 0,5 , che è la quantità di
mele marce per confezione da 5. Sebbene questo risultato abbia un’interpretazione fisica un po’
traballante (le mezze mele non sono previste dal nostro modello), l’interpretazione matematica è
chiarissima e si tratta proprio del valore che ci attendevamo. In generale moltiplicando fra loro le
probabilità di certi eventi legati a valori numerici per il valore stesso, otteniamo un specie di
media statistica, il cosiddetto valore atteso.
Definizione
Si chiama valore atteso v (chiamato anche aspettazione o media matematica) di una
variabile casuale x , la somma di tutti i valori che la variabile può assumere, ciascuno
moltiplicato per la probabilità di essere assunto.
In simboli il valore atteso si può esprimere così: se ad ogni evento possibile Ei associamo un
valore xi , allora il valore atteso di x è dato dalla formula v[ x] = x1 ⋅ P( E1 ) + x2 ⋅ P( E2 ) + ... .
Nel caso in cui gli eventi possibili siano n la formula diventa v[ x ] =
n
∑ ( x ⋅ P( E ) ) .
i
i
i =1
Il valore atteso gioca un ruolo fondamentale nel calcolo dei giochi d’azzardo: se il valore atteso è
negativo (cioè in media si perdono più soldi di quanti non sene vincano) il gioco si dice iniquo.
Ogni buon matematico dovrebbe guardarsi bene dal perdere soldi in un modo così sciocco.
Esempio: La roulette
Supponiamo di giocare alla roulette puntando una certa somma fissa S . Le regole prevedono
che i soldi gettati sul tavolo verde S spettino sempre al banco e che in caso di vittoria il Casinò
paghi 36 volte la somma giocata, cioè restituisca 36 ⋅ S . Calcoliamo ora il valore atteso, cioè :
probabilità di perdere × ( − S ) + probabilità di vincere × ( 36 S ) . I numeri che possono uscire
sono 37 (lo 0 e i numeri fra 1 e 36), per cui
P ( vittoria ) =
1
. La probabilità di perdere la
37
somma giocata S è 1, si tratta infatti di un evento sicuro perché se non si mettono i soldi sul
banco non si può giocare. Trattandosi di una perdita, nel calcolo del valore atteso bisognerà
utilizzare − S .
Riassumendo abbiamo che il valore atteso è v = 1⋅ ( − S ) +
71
1
−37 + 36
1
⋅ ( 36 ⋅ S ) = S ⋅
= −S ⋅ .
37
37
37
Abbiamo quindi dimostrato che in media il giocatore perde
1
dei soldi giocati. Può non
37
sembrare molto, ma se in una serata su un certo tavolo verde “gira” 1 milione di euro (cifra
tutt’altro che peregrina) il casinò incassa in media ca. 27.000 euro. A tavolo!
Esercizio: Il gioco del Lotto
Le regole del gioco del Lotto sono le seguenti: da una ruota di 90 numeri (da 1 a 90) vengono
estratti 5 numeri diversi. Esistono 5 diversi tipi di pronostico: si può giocare un solo numero
(detto estratto semplice), due numeri (ambo) e così via fino a 5 (rispettivamente terno, quaterna,
cinquina) puntando una certa somma S . Si vince se e solo se tutti i numeri giocati sono stati
effettivamente estratti. A seconda del tipo di pronostico sono previsti i seguenti premi:
Tipo di pronostico
(quantità di numeri giocati)
Vincita
Estratto semplice (1)
11,232 ⋅ S
Ambo (2)
Terno (3)
Quaterna (4)
Cinquina (5)
250 ⋅ S
4500 ⋅ S
120.000 ⋅ S
6.000.000 ⋅ S
Dimostra che il gioco del Lotto è un gioco iniquo.
Soluzione: Il gioco del Lotto
Evidentemente ogni cinquina ha la stessa probabilità di essere estratta e quindi possiamo rifarci
alla definizione di probabilità P( E ) =
numero di casi favorevoli
. Visto che l’ordine di estrazione è
numero di casi possibili
 90 
 (questi implica che dovremo considerare cinquine non
5
irrilevante i casi possibili sono 
ordinate anche quando conteremo i casi favorevoli).
Supponiamo ora di giocare un certo numero e chiediamoci quante cinquine (non ordinate) lo
contengano. Fissando il numero da noi giocato ed “estraendo” a caso altri 4 numeri siamo in
grado di formare esattamente le cinquine che dobbiamo considerare. Questo vuol dire che
 89 
 casi favorevoli.
4
esistono 
 89 
 
4
89 ⋅ 88 ⋅ 87 ⋅ 86 ⋅1 ⋅ 2 ⋅ 3 ⋅ 4 ⋅ 5
5
1
In definitiva P (estratto semplice) =   =
=
= ≅ 0, 0556 .
 90  90 ⋅ 89 ⋅ 88 ⋅ 87 ⋅ 86 ⋅1 ⋅ 2 ⋅ 3 ⋅ 4 90 18
 
5
72
Ragionando in modo analogo possiamo dire che il numero delle cinquine che contengono un
 88 
 
3
 88 
2
due numeri fissati è   e quindi P ( ambo) =   =
.
 90  801
3
 
5
Ormai
dovremmo
aver
capito
la
regola
generale
e
quindi
calcoliamo
subito
 87 
 86 
 85 
 
 
 
2
1
0
1
1
1
P(terno) =   =
, P ( quaterna ) =   =
e P (cinquina ) =   =
.
 90  11.748
 90  511.038
 90  43.949.268
 
 
 
5
5
5
Notiamo che nell’ultimo caso (quello relativo alla cinquina) nel numeratore compare il termine
 85 
  , cioè 1. Questo non è sorprendente perché giocando una cinquina esiste una ed una sola
0
cinquina che la contiene e cioè essa stessa (ricordiamo che non stiamo considerando l’ordine).
Riportiamo i valori da noi trovati nella seguente tabella. Le ultime due colonne si riferiscono alla
perdita, che corrisponde evidentemente alla somma investita (l’evento è certo perché per
giocare bisogna necessariamente pagare).
Tipo di pronostico
(quantità di numeri giocati)
Estratto semplice (1)
Probabilità di
vincere
Vincita
1
18
2
801
1
11.748
1
511.038
1
43.949.268
11,232 ⋅ S
Ambo (2)
250 ⋅ S
Terno (3)
4500 ⋅ S
Quaterna (4)
120.000 ⋅ S
Cinquina (5)
6.000.000 ⋅ S
Perdita
Probabilità
di perdere
S
1
S
1
S
1
S
1
S
1
Assumendo che la perdita equivalga ad un guadagno negativo, calcoliamo il valore atteso nei
rispettivi casi. Ovviamente v(Gi ) = Vi ⋅ P(Vi ) + Zi ⋅ P(Zi ) , dove Gi indica il guadagno del rispettivo
tipo di gioco (estratto semplice, ambo, ecc.), P(Vi ) la probabilità di vincere e Vi l’ammontare
della vincita, P(Zi ) la probabilità di perdere e Z i l’ammontare della perdita (sempre con il
segno negativo).
Tipo di pronostico
Calcolo del valore atteso
(quantità di numeri giocati)
Estratto semplice (1)
11, 232 ⋅ S ⋅
1
18
 11, 232
+ (− S ) ⋅ 1 = S 
 18
73


− 1
Valore atteso
(approssimato)
−0,376 ⋅ S
2
Ambo (2)
250 ⋅ S ⋅
Terno (3)
4500 ⋅ S ⋅
Quaterna (4)
120000 ⋅ S ⋅
Cinquina (5)
6000000 ⋅ S ⋅
801
 500 
− 1
 801 
+ (−S ) ⋅1 = S 
1
11748
 4500 
− 1
 11748 
+ (− S ) ⋅1 = S 
1
511038
 120000 
− 1
 511038 
+ (− S ) ⋅1 = S 
1
43949268
 6000000 
−1
 43949268 
+ ( −S ) ⋅1 = S 
−0,376 ⋅ S
−0,617 ⋅ S
−0,765 ⋅ S
−0,863⋅ S
Osserviamo che non è possibile calcolare il valore atteso del Lotto complessivamente, perché
esso consiste in verità di 5 giochi diversi. Nonostante questo limite possiamo asserire che
nessuno dei cinque giochi è equo e quindi non lo è nemmeno il Lotto in quanto tale. Per perdere
soldi più velocemente si consiglia comunque di giocare cinquine secche.
Problema 4.1 - 7 volta testa, 3 volte croce
Calcolare la probabilità che, lanciando una moneta per 10 volte, esca 3 volte croce e 7 volte
testaM
a) in un ordine qualsiasi
b) con il vincolo che ogni lancio sia preceduto o seguito (non necessariamente entrambi)
da un lancio con lo stesso esito.
Problema 4.2 - 10 dadi, 4 “1”
Calcolare la probabilità che, lanciando un dado per 10 volte, esca esattamente per 4 volte “1”.
Problema 4.3 - 8 con due dadi
Con che probabilità lanciando due dadi e sommando i risultati si ottiene 8?
Problema 4.4 - Compleanni
Con che probabilità almeno due alunni di una classe composta da 23 ragazzi nati nel 1996
hanno il compleanno lo stesso giorno?
Problema 4.5 - Almeno due assi a Poker
Un giocatore di poker riceve all’inizio del gioco cinque carte da unmazzo di 52 (l’insieme delle
52 carte si chiama mano). Qual è la probabilità di ricevere almeno 2 assi?
74
• Soluzioni dei problemi della Sezione 4
Soluzione 4.1 - 7 volta testa, 3 volte croce
a) Il problema è evidentemente un esperimento di Bernoulli con p =
3
1
, n = 10 , k = 3 (o k = 7 ).
2
7
 10   1   1  10 ⋅ 9 ⋅ 8 1
120
Il risultato è quindi P =   ⋅   ⋅   =
⋅ 10 =
≅ 0,117 = 11,7% .
1⋅ 2 ⋅ 3 2
1024
 3  2 2
b) Notiamo che il vincolo posto non è altro che un modo complicato per dire che gli esiti dello
stesso tipo devono essere “raggruppati”. Ci interessano quindi i due esiti (ordinati):
(C , C , C , T , T , T , T , T , T , T ) e (T , T , T , T , T , T , T , C , C , C ) . Visto che ogni esito ha la stessa
3 volte
7 volte
probabilità p =
7 volte
3 volte
1
1
1
il risultato è 2 ⋅ 10 = 9 ≅ 0,00195 ≅ 0,2%
10
2
2
2
Soluzione 4.2 - 10 dadi, 4 “1”
Anche in questo caso si tratta di un esperimento di Bernoulli. Per ogni lancio la probabilità che
1
e
quindi,
applicando
6
4
6
 10   1   5  10 ⋅ 9 ⋅ 8 ⋅ 7 56
P =  ⋅  ⋅  =
⋅
≅ 0,05427 ≅ 5,4% .
1 ⋅ 2 ⋅ 3 ⋅ 4 610
 4  6 6
esca
“1”
è
p=
la
formula,
otteniamo
Soluzione 4.3 - 8 con due dadi
In un lancio di due dadi esistono 36 risultati diversi (considerando rilevante l’ordine) fra loro
(2,6) , (3,5) , (4,4) , (5,3) e (6, 2) . La
equiprobabili. I casi favorevoli sono i seguenti:
probabilità richiesta è quindi
5
≅ 0,139 = 13,9% .
36
Soluzione 4.4 - Compleanni
Visto che il 1996 è stato un anno bisestile ci sono 366 possibili compleanni. Il calcolo diventa
decisamente più semplice se si risponde preliminarmente alla domanda inversa: qual è la
probabilità che in una classe di 23 alunni nessuno abbia la stessa data di nascita di un altro?
Ragioniamo per passi:
Prendiamo l’alunno 1 e mettiamolo da parte. Prendiamo poi l’alunno 2 e calcoliamo la
probabilità che non faccia gli anni lo stesso giorno dell’alunno 1. Risulta ovviamente
P1 =
numero compleanni diversi
365
=
.
numero compleanni possibili 366
Messo da parte anche l’alunno 2, la probabilità che l’alunno 3 abbia un compleanno diverso dai
primi due è
P2 =
numero compleanni diversi
364
=
.
numero compleanni possibili 366
Proseguendo così otteniamo
75
numero compleanni diversi
363
numero compleanni diversi
362
=
P4 =
=
,
numero compleanni possibili 366
numero compleanni possibili 366
numero compleanni diversi
366 − n
=
generale Pn =
.
numero compleanni possibili
366
P3 =
e
in
Notiamo che in una classe di 23 alunni l’ultima probabilità da calcolare è P22 e non P23 (il primo
alunno infatti è stato messo da parte subito e non ha contribuito al calcolo).
La probabilità complessiva è data dal prodotto delle singole probabilità (pensiamo come al
solito all’albero) e quindi P = P1 ⋅ P2 ⋅ P3 ⋅ ... ⋅ P21 ⋅ P22 =
calcolo
tale
365 364 363
344
. Effettuando il
⋅
⋅
⋅ ... ⋅
366 366 366
366
valore
risulta
3237442261911402473879948207367308926540124734375
≅ 0,493677
7488607914347662661050955694294045102373250976162
essere
cioè
suppergiù
49,4% .
Ricordiamoci ora che abbiamo calcolato il problema inverso. La risposta alla prima domanda,
cioè “Con che probabilità almeno due alunni di una classe di 23 studenti fanno gli anni lo stesso
giorno?” è allora 100% − 49,4% = 50,6% , cioè più del 50% .
Soluzione 4.5 - Almeno due assi a Poker
Questo tipo di problema permette una schematizzazione semplice se supponiamo che le carte
in mano siano ordinate in un certo modo. Visto che noi “imponiamo” un ordine, consideriamo
tutte le mani composte dalle stesse carte ma mischiate in modo diverso fra loro equivalenti,
cioè stiamo considerando cinquine non ordinate. Naturalmente ogni mano è equiprobabile e
quindi possiamo ricondurre il calcolo al rapporto fra casi (mani) favorevoli e casi (mani)
 52  52 ⋅ 51 ⋅ 50 ⋅ 49 ⋅ 48
.
=
1⋅ 2 ⋅ 3 ⋅ 4 ⋅ 5
5
possibili. Per quanto appena detto le possibili mani sono 
Notiamo che nel problema si parla di “almeno due assi” e quindi si può trattare anche di un tris
 4
 2
o di un poker d’assi. Esistono in tutto   coppie d’assi (il numero di coppie non ordinate in un
insieme di 4 elementi). Le restanti carte sono una tripletta non ordinata composta da carte
 50 
 diverse alternative.
3
diverse dalle prime. Visto che le carte rimanenti sono 50 esistono 
 4   50 
 ⋅ 
2
3
Riassumendo abbiamo una probabilità di     di avere almeno una coppia d’assi servita,
 52 
 
5
4 ⋅ 3 50 ⋅ 49 ⋅ 48
1⋅ 2 ⋅ 3 ⋅ 4 ⋅ 5
4 ⋅3⋅ 4 ⋅5
cioè
⋅
⋅
=
≅ 0.0452 ≅ 4,5%
1 ⋅ 2 1 ⋅ 2 ⋅ 3 52 ⋅ 51 ⋅ 50 ⋅ 49 ⋅ 48 2 ⋅ 52 ⋅ 51
76
Scarica